Você está na página 1de 17

MEDICINE

FACULTY OF MEDICINE -COLOMBO


2003 A/L DECEMBER-2009

FINAL PAST
PAPERS
WITH
COMMON MCQs

THIS BOOK IS DISTRIBUTED FREELY BY THE


MSWS AND MFSU FOR THE BENEFIT OF THE
STUDENTS OF 2005 A/L BATCH

T/F
SLE - episcleritis
Ankylosing spondylitis - iritis
Rheumatoid arthritis - Atlanto axial subluxation
SLE - Apical pulmonary fibrosis
Hypothyroidism
Grave's disease when treated with Carbimazole,
remission is rare
Diffuse goitre is treated with subtotal thyroidectomy.
Radioactive Iodine is contra indicated in pregnancy.
Carbimazole, when given in multi nodular goitre
commonly induce a remission.
For sub acute thyroiditis, beta blocker therapy can be
used alone.
Following infections and carcinomas are correctly
matched.
HIV - Kaposi's sarcoma
EBV - Burkit's lymphoma
CMV - Colonic carcinoma
Hepatitis C - Hepatocellular carcinoma
Helicobacter pylori - Gastric carcinoma
Regarding complete heart block
Irregular cannon a waves
Dissociation of waves and QRS complexes in ECG
Varying intensity of 1st heart sound
Mid diastolic murmur at apex
Syncopal attacks as presenting complaint
60 year old patient presents with right sided
hemipariesis. Management of the 1st 24hr of this
patient includes
Clopidogrel + Aspirin combination
Contrast enhanced CT brain
Phenytoin
5% dextrose IV
ECG with rhythm strip
T/F Regarding portal hypertension
Thrombocytopenia is a common sequale
Extrahepatic causes are common in elderly
Splenomegaly is a cardinal physical sign
Venous hum can heard over the abdomen
Hepatic vein thrombosis is a cause
Which of the following predisposes to GORD
Smoking
Pregnancy
Metochlopromide
Systemic sclerosis
Achalasia cardia
Abnormalities seen in CRF

A PROJECT OF MSWS AND MFSU- COLOMBO (2010/2011)

10)

11)

12)

13)

14)

Intravascular volume contraction


Hyperphosphataemia
Hyperuricaemia
Hypertriglyceridaemia
Hypernatremia
Regarding ARF which of the following are correctly
matched
RBC cast -AGN
Field full pus cells - Pyelonephritis
Microscopic haematuria -Leptospirosis
Jaundice/ bleeding tendency -Hepatorenal syndrome
Clinical and biochemical features of minimal change
disease
Proteinuria >3g/24hrs
Gross oedema
Microscopic haematuria
High LDL cholesterol
Venous thrombosis
Regarding Narcolepsy
Sleep paralysis
Cataplexy
Uncontrollable sleep episodes
Hypnagogic hallucinations
PCV
Measured by the capillary method is higher than the
venous value.
Is elevated in babies born with IUGR
Is a good indicator of acute blood loss
Is expressed as a percentage
Drops in the recovery phase of DHF
S. osmolality 350 (increased) and u. osmolality is
200. What may be the possibilities?
Cranial DI
Nephrogenic DI
DM
Primary polydipsia
CRF
Leukoerythroblastic response seen in
Severe sepsis
Cytotoxic drugs
Severe haemolysis
Prostatic CA
Viral infection

1 | Page

MEDICINE2004 A/L- DEC 2010 COLOMBO


1.
2.
3.
4.
5.
6.
7.
8.
1.
2.
3.
4.
5.
9.
1.
2.
3.
4.
5.
10.
1.
2.
3.
4.
5.
11.
1.
2.
3.
4.
5.
12.
1.
2.
3.
4.
5.
13.
1.
2.

Chronic diarrhea
Flare up of SLE is suggestive of,
Effective in reducing partial pressure
Regarding chronic Hepatitis,
Recognized feature of Adult Polycystic
Kidney disease,
Infective Endocarditis,
TH,
Anti depressants are given because of
their,
Anti cholinergic effect
Produce sedation
Improve sleep
Reduce anxiety
Mono amine receptor inhibition
Following are normal experiences,
Night terrors
De javu phenomenon
Delusion of perception
Perservation
Hypnogogic hallucinations
True / false
Olanzapine has more extrapyramidal
side effects than haloperidol
Trifluperazine contra-indicated with
anti-epileptic drugs
Fluoxetin shouldnt be given in
obesity
Chordiazepoxide
is
used
in
management of delirium tremens
Imipramine is useful in the
management of nocturnal enuresis
Organophosphate poisoning,
Anticholinesterase inhibited
Gastric lavage is indicated 4 hours
after ingestion
Atropine should be repeated till lungs
become clear
Atropine is effective in controlling
seizures
Pralidoxin act by reversing the
inhibition of Ach
Feature of Hump nosed viper envenoming,
Coagulopathy
Neurotoxicity
Nephrotoxicity
Blisters
Myositis
Vesicular eruptions are seen in,
Varicella zoster
Herpes simplex

Impetigo
Tinea corporis
Cutaneous larva migrans
14.

22.

HIV,
Screening test is PCR
Seroconversion within 6 months
Written informed consent before
confirmatory test

15.

16.

17.

18.
19.

20.

21.

IgG in 3 months active infection


High CD4 count high viral load
Diagnosis of Pulmonary TB
Early morning sputum sample is not
used to diagnose AFB
Sputum should be collected to a
sterile container for culture
Can be excluded when mantoux
negative
Detecting DNA in sputum is the best
method
Bronchoscopy aspiration is not
helpful
Giardiasis,
Travelers diarrhea
A cause for Folic acid deficiency
Cause blood and mucus diarrhea
Weight loss
Tinidazole
True/ False
Diabetes mellitus is caused by
osmotically active particles
Reduced plasma osmolality and
increased urine osmolality Primary polydipsia
Anaemia with increased unconjugated bilirubin with
reticulocytosis
Hereditary spherocytosis
Anaemia of chronic disease
Treating the underline cause will
improve
Majority MCV is reduced
Body iron is reduced
TIBC reduced
Reticulocytes increased
Pre renal more than renal in ARF,
Blood pressure 80/60 mmHg
Bladder empty
Clinical
evidence
of
severe
dehydration
Concentrated urine
Serum K+ 6.2
Nephritic due to minimal change is
unlikely if,
Hypertension
Ascites
Haematuria

23.

24.

25.

26.

27.

28.

29.

Thrombosis
Hyperlipidaemia
Recognized cause of Metabolic Acidosis
with increase anion gap,
Aspirin
DKA
Starvation
Paracetamol poisoning
Diarrhea
Tight blood glucose control with insulin
in DM results in
Reduced incidence of retinopathy
Increased hypoglycemic events.
Reversal of urinary protein excretion?
Poor long term prognosis in type 2
DM
Weight loss
Leptospirosis,
Severe myalgia is a early feature
Microscopic haematuria is seen
Prothombin time is increased
Myocarditis lead to circulatory
collapse
Benzathine penicillin is the drug of
choice
Peripheral blood macrocytosis is seen in
Untreated Fe deficiency anaemia
Folate deficiency
Aplastic anaemia
Thalassaemia major
Megaloblastic anaemia
Regarding OCD
Flooding is used in treatment
BZD helpful to control rituals
Thought stopping effective in
treatment
Relatives are asked continuously to
reassure
Complications/
features
of
hypothyroidism
Cerebellar ataxia
Ascites
Hyponatraemia
Carpel tunnel syndrome
Drugs contra indicated in liver failure
Isopane insulin
Pioglitazone
Metformin
Gliclazide
Acarbose
Metabolic alkalosis
Vomiting
Starvation
Frusemide

A PROJECT OF MSWS AND MFSU- COLOMBO (2010/2011)

CRF
Aspirin poisoning
31. WOTF investigations help to find the cause in 25
year old man with hypertension,
ECG
Abdominal USS
CXR
32. COPD management,
High oxygen concentration
IV frusemide
33. Severity of pneumonia indicated by,
Leucopenia
Temp. 38C
WBC count 15,000/mm3
34. Legionella infection,
Most with those who work outside
Initially similar to viral flu
Rifampicin used in severe infection
35. WOTF is/are used to analyze non- parametric data?
Spearman analysis
Analysis of variance
Student T test
Multiple regression analysis
Mann W..... U test

JAFFNA- AUGUST 2010


MEDICINE
1. Acute Nephritic syndrome is characterized by
a.
DM nephropathy
b.
Minimal change disease
c.
Amyloidosis
d.
SLE
e.
HSP
2. Lung Abscess
a.
Results from polymicrobial infection
b.
Known to complicate bronchial CA
c.
Caused by infective endocarditis of
tricuspid valve
d.
Effectively treated with antibiotics for
two weeks
e.
Alternatively treated with appropriate
resection of lung tissue
3. Complications of aortic valve endocarditis
a.
Pulmonary embolism
b.
Pericarditis
c.
Acute aortic regurgitation
d.
Renal infarction
e.
Complete heart block
4. Person attempted suicide regarded on having
high risk if person is
a.
Living alone
b.
Abusing alcohol
c.
Leaving a suicide note

2 | Page

d.
e.

Has history
attempts
Employed

of

previous

similar

5. EBV
a.
b.
c.
d.
e.

Causes immune haemolytic anemia


Treated with acyclovir
Predispose to lympho proliferative
disease
Causes hairy leukoplakia in HIV patient
Confirmed by lymph node biopsy

6. Causes of delirium include


a.
Severe infection
b.
Increase thiamine
c.
Hypoglycaemic
d.
Abrupt withdrawal of lithium
e.
Severe stress
7. Management of snake bite
a.
Haematuria is an indication for
immediate antevenom
b.
Krait bite with respiratory failure
treatment with antevenom is adequate
c.
Tetanus toxoid should be given
d.
Antevenom contraindication in patient
give allergy history
e.
Patient with bite caused by identified
non venomous snake need not be
admitted to hospital.
8. Following are features of multiple myeloma
a.
a. Hyper viscosity
b.
Pancytopenia
c.
Elevated alkaline phosphatase
d.
Reduced serum calcium level
e.
Osteosclerotic bone lesions
9. Cause of ST depression
a.
Digoxin poisoning
b.
LVH
c.
Left ventricular aneurysm
d.
Unstable angina
e.
Pericarditis
10. Following are X- link dominant
a.
Vit D resistant rickets
b.
Hemophilia B
c.
Pyruvate kinase deficit
d.
Achondroplasia
e.
Wilson disease
11. A 55 years old previously well woman
presented with profuse diarrhea of and
decrease UOP 3 day duration. She likely to
a.
PR > 100 bpm
b.
b.peripheries
are
warm
and
vasodilated
c.
Arterial blood pH < 7.2
d.
RBC cast in the urine

e.

Response better to 5% dextrose than


0.9 NS
12. Causes of delirium include
a.
Severe infection
b.
Increase thiamine
c.
Hypoglycaemia
d.
Abrupt withdrawal of lithium
e.
Severe stress
13. Complete heart block
a.
Widen pulse pressure
b.
Produce cannon wave
c.
Regular ventricular rate
d.
Associated with atrial fibrillation
produced irregularly irregular pulse
e.
Mostly due to inferior MI than anterior
MI
14. Obesity seen
a.
Cushings syndrome
b.
PCOS
c.
Addisons
d.
Graves
e.
Prolactinoma
15. Regarding ARF
a.
Hamodialysis is mandatory
b.
Spontaneous resolution occurs when
due to ATN
c.
Ca Gluconate IV promotes recovery
d.
Patient should be confined to bed rest
e.
NSAID is recommended for pain relief
16. Recognized manifestation of acute nephritis
( AGN in)
a.
Diabetic nephropathy
b.
Amyloidosis
c.
HSP
d.
SLE
e.
Minimal change disease
17. Causes of chronic diarrhea
a.
H. Pylori infection
b.
Rota virus
c.
Metformin therapy
d.
Carcinoid syndrome
e.
Lactose intolerance
18. Leptospirosis
a.
Is caused by gram (+) ve bacilli
b.
Causes thrombocytopenia
c.
Causes ill defined lung shadows in
CXR
d.
Is best treated with gentamycin
e.
Causes aseptic meningitis
19. Leptospirosis
a)
Is caused by Gram ( +)ve bacilli
b)
Causes thrombocytopenia

c)

Causes ill defined lung shadows in


CXR
d)
Is best treated with gentamycin
e)
Causes aseptic meningitis
20. Regarding fungal infection of the skin
a)
Scaly patches with central clearing is
seen in tinea corporis
b)
Satellite lesions are seen in tinea cruris
c)
Dermatophyte infections spread by
direct contact
d)
A fungal skin infection that has been
modified by topical steroid application
is called tinea incognito
e)
Oral itraconazol is the drug of choice in
tinea capitis
21. In treatment of hypothyroidism
a)
Carbimazol is contraindicated in
pregnant patient
b)
Radioactive iodine treatment is not
recommended in graves disease
c)
Radioactive iodine treatment is
effectively monitored by estimation of
TSH level in blood
d)
Radio-active Carbimazol is preferred to
Carbimazol in toxic multi nodular goiter
e)
With Carbimazol therapy monitoring of
WBC cannot use to detect neutropenia
22. Loss of tendon reflexes characteristic of
a)
Motor neuron disease
b)
Vit B12 deficiency
c)
GBS
d)
Myasthenia Gravis
e)
Transverse myelitis
23. ST depression seen in
a)
Pericarditis
b)
Digoxin treatment
c)
Unstable angina
d)
LVH
e)
LV aneurysm
24. Septic Arthritis
a)
Commonly produced by gram (-) than
gram (+) bacteria
b)
Involve multiple joints
c)
Characterize by painful joint a limitation
of movements
d)
Accurately diagnosed by joint
radiography
e)
Effectively treated with intra articular
antibiotics
25. Correctly paired
a)
People in the street talking about meGrandiose delusion

A PROJECT OF MSWS AND MFSU- COLOMBO (2010/2011)

b)

I hear a voice describing my actionsthought echo


c)
My next door naighbour is planning to
kill me- delusion of reference
d)
I hear voices which order me not to
eat- 3rd person auditory hallucination
e)
My thoughts known to others- thought
broadcasting
26. 26yr old boy diagnosed as DM, it is more likely
to type 2 than type 1 if he has,
a)
Strong family history of DM
b)
BMI-26
c)
Rapid loss of 5kg at time of
presentation
d)
Ketoacidocis on presentation
e)
Acanthosis nigricans
27. 17yr old 50kg previously healthy man admitted
with drowsiness on examination, froth in the
mouth, PR-50bpm, RR-80pm, Bp-70/50mmHg,
GCS8/15, lung aus-diffuse crepitation, pupilpinpoint. Following are T/F
a)
Gastric lavage done immediately
b)
History
suggestive
an
organophosphate ingestion
c)
0.9% normal saline should be started
d)
IV atropine bolus should be given
e)
Should be intubated immediately
28. Myocarditis T/F
a)
Specific ECG changes are present
b)
Sudden cardiac arrest occur in young
adult
c)
Viral infection is a known cause
d)
Troponin are (+)
e)
Self limiting in majority
29. Regarding DM nephropathy
a)
Absence of protein in heat coagulation
test exclude DM nephropathy
b)
ACEI uses delay the progression of
illness
c)
Most of patient need biopsy for
diagnosis
d)
Blood urea nitrogen irreversibly
elevated
e)
Good glycemic control delay the onset
of nephropathy
30. in HIV patients
a)
Oesophageal candidiasis
b)
Toxoplasmosis of brain
c)
Herpes zoster in multiple dermatomes
d)
Atypical mycobacterial infection
e)
Kaposi sarcoma
31. Potent reversible causes of cognitive
impairment
a)
Lewy body disease

3 | Page

b)
Hypothyroidism
c)
Hyponatraemia
d)
Hepatic encephalopathy
e)
Parkinsons disease
32. in the treatment of ascites due to cirrhosis
a)
Bed rest is mandatory
b)
Salt restriction is the principle cure of
hyponatraemia
c)
Use of frusemide is known to worsen
hepatic encephalopathy
d)
FFP given a better therapeutic
response than stored plasma
e)
Large volume abdominal paracentesis
should be combined with albumin
infusion
33) Regarding rheumatoid arthritis
a)
Distal interphalangeal joint involvement
is a characteristic feature
b)
Restricted pattern of lung function is
due to lung parachyal disease
c)
Thrombocytosis indicate active disease
d)
Episcleritis is complication
e)
Pitting of nail is characteristic
manifestation
34) 70 year old male presented with tremor &
difficulty in walking which favour Parkinsonism
a)
Intention tremor
b)
Unilateral tremor
c)
Predominately affects the lower limb
d)
Broad gait base
e)
Rapid response to levodopa
35) Regarding pulmonary TB
a)
BCG gives protection from post
primary TB
b)
Milliary TB treated for 6 months
c)
Sputum positive after 2 months of
treatment highly suggestive of resistant
organism
36) 28 year old gentlemen presented dengue for 4
days duration .Coetaneous features cold &
clammy peripheries.PR 110/min, Bp90/80
Hgmm.
Sx
platelets-30,000,Hb14g/dl,leucopenia,PCV -52%
a)
Presently initially resuscitate with 40%
dextran
b)
Initial infusion rate 5ml/kg/hr
c)
Platelet transfusion is indicated
d)
Increasement of PCV due to leakage
e)
After 4 hours PCV -45%,condition
remain same following resuscitation
with normal saline
37) Features associated with high mortality in
pneumonia include
a)
PR>30min

b)
PR of 90/min
c)
BU of 17 mmol/L
d)
Mental confusion
e)
Blood culture positive
38) A newly married female of 20yrs presents with
dysuria7, frequency of micturition & suprapubic
pain. WO T/F
a)
Amoxicillin is effective with treatment
b)
Needs extensive evaluation of the
urinary tract for structural abnormality
c)
Treated with vaginal oestrogen cream
d)
Alkaline urine suggestive of infection
with proteus
e)
The patients sexual partner should be
investigated
39) Following are found in hyperactivity disorders
a)
Its onset may occur before 36 months
b)
Equal in male & female
c)
Hyperactivity found only in home
d)
Co- morbidity associated with
hyperactivity disorders
e)
Methylphendylate is useful in treatment
40) T/F DHF
a)
It does not occur in the age of 6/12
b)
A negative Hess test exclude the
diagnosis
c)
Bradycardia acu convalescence
d)
Hyperosmolar fluid are preferable to
Iso-osmolar fluid in shock
e)
DIC is a complication
41) In Acute lymphoblastic leukaemia
a)
Most tumor cell derive from B cell line
b)
CNS involvement indicates poor
prognosis
c)
WBC count more than 50x107 /L
indicates poor prognosis
d)
Monoarthritis is a feature
e)
CXR shows mediastinal mass

OBSTETRICS AND GYNAECOLOGY


FACULTY OF MEDICINE COLOMBO
DECEMBER-2009
Management of PPROM
Vaginal examination is done to diagnose
CRP to exclude chorioamnionitis
Tocolytics until fetal viability reached
Antibiotics
Dexamethasone is started
Group B streptococcal in pregnancy
10% of mothers have the infection

10)

11)

Vaginal delivery is contraindicated


Causes puerperal sepsis
Associated with congenital cardiac anomalies
Cephalosporin is the preferred drug
Features of obstructed labour
Maternal tachycardia seen
Bandel's rings
Separation of fetal skull bones
Haemorrhoids seen
Oedematous lower genital tract seen
IUGR
>3kg birth weight excludes IUGR
Pre-eclampsia causes symmetrical IUGR
Olighydramnios occur due to placental insufficiency
Physiological changes during pregnancy
Reduced respiratory rate
Increased pulse rate
Increased GFR
Increased haematocrit
Increased renal threshold for glucose
Baby of a diabetic mother
Hyperglycemia
Hypothermia
Hypocalcemia
Retinopathy
Jaundice
Endometriosis
May present with worsening of 1rydysmenorrhoea
Highly resistant to medical management
Caused by retrograde menstruation
Causes early pregnancy loss
Treated with Danazole
Regarding leiomyosarcoma
Associated with parity
Mainly haematogenous spread
5 year survival is equal to endometrial carcinoma
Arises commonly from benign lesions
Commonly causes post menopausal bleeding
Regarding Cu T 380
Prevents implantation
Replaced every 2 years
Can be used as an emergency contraceptive method
Causes heavy menstrual bleeding
Contraindicated in active PID
An otherwise healthy primigravida had dilatation
from 4cm to 6cm in 8 hours. What interventions can be
done?
Pain relief
Improve hydration
Asses the position of the head
Ergometrine IV infusion
Assess uterine contractions
Increased prolactin levels seen in
Hypothyroidism

A PROJECT OF MSWS AND MFSU- COLOMBO (2010/2011)

12)

13)

14)

15)

16)

17)

18)

19)

Bronchial asthma
Meningioma
Pituitary neoplasm
Tri cyclic antidepressants
Premalignant lesion of cervix
Can be visualized by naked eye examination
Is symptomatic
Smoking is a risk factor
Colposcopy directed biopsy useful for diagnosis
Falls under 1 a of FIGO classification
Gestational trophoblastic disease
Common in nulliparity
Is associate with poor prognosis if preceding pregnancy
deliver at term
Is treated with Metronidazole alone in recurrent disease
Diagnosed by USS
Associated with Triploidy
Germ cell tumours of ovary include
Dysgerminoma
Granulosa cell tumour
Androblastoma
York sac tumour
Adenofibroma
Regarding SFA
Sperm count of 25-30 million/ml need further
investigation
Active progressive motility of less than 10% is abnormal
Teratospermia is abnormal sperm
Should dp done only when there is no abnormality in
female
Abstinence of sexual intercourse for 3 days, need prior
to investigation
Trichomonas
Male will not get the infection because it is washed
away in urine
Treated with metronidazole 1g single dose
Male partner treatment is essential
Diagnosed by wet saline mount of posterior fornix
Chlamydia in female
Foul smelling vaginal discharge
Causes tubal infertility
Causes PID
Male partner need not to be treated
Treated with Doxycycline
Endometriosis
Recognized complication of 1rydysmenorrhea
Causes recurrent miscarriages
Causes tubal infertility
Due to retrograde flow of menstruation
18 year, phenotypically female, not menstruated, no
breast development, high FSH and LH, low oestradiol.
Whatare the management required
Karyotyping
Trans abdominal USS

4 | Page

c)
d)
e)
20)
a)
b)
c)
d)
e)
21)
a)
b)
c)
d)
e)
22)
a)
b)
c)
d)
e)
23)
a)
b)
c)
d)
e)
24)
a)
b)
c)
d)
25)
a)
b)
c)
d)
e)
26)
a)
b)
c)
d)
e)
27)
a)
b)
c)
d)
e)

X ray kidney
28)
Regarding urinary incontinence
Reassurance about fertility
Cystourethrocoele is a cause
Oestradiol therapy
GSI is commoner than urge incomtiiinnco
Complications of induction of labour includes
Surgery is preferable for GSI
Cord prolapsed
Pelvic floor exercises are useful in the management
Placental abruption
Oxybutalin for detruesor instability
Hypernatremia
29)
Regarding estrogen
Prematurity
Plays a role in epiphyseal closure of long bones
Increased
incidence
of
unconjugated
If given to women with ovarian agenesis results in
hyperbilirubinaemia
growth of breast tissue
Causes of transverse lie includes
Estrone is predominate inmenopause
Placenta praevia
Mainymetabolised in kidney
Fibroids
Is teratogenic
IUGR
30)
Drug treatment in management of PPH
Oligohydramnios
Ergometrine 0.5mg IM
Grand multipara
Synto 5U IV
Regarding OP position in pregnancy
IV synto 20U in 500 ml N/S
It occurs in 50% early labour
Intramyometrial inj. Of prostaglandin E2
Majority changes to OA position
Pervaginal PG E2
Majority needs assisted vaginal delivery
st
Doesn't causes prolonged 1 stage of labour
OBSTETRICS AND GYNAECOLOGY
Causes deep transverse arrest
FACULTY OF MEDICINE COLOMBO
PPH due to genital tract trauma
DECEMBER 2010
Cervical tears are treated by packing the vagina
Uterus is hard
Complications of premature birth
OP position is more related to the trauma than others
Hyperviscosity
Wait till 500ml of blood loss
Hypothermia
Cervical tears are treated by IV ergometrine
Intraventricular haemorrhage
Bleeding in early pregnancy
Resporatory distress
Indication of high risk for pre eclampsia in late
Jaundice
pregnancy
Hepatitis A infection in pregnancy causes,
Doppler is indicated
Causes congenital anomalies
Better assed by trans-vaginal USS
Better prognosis than Hep. A
Treated with vaginal pesseries
Chronic carrier state of neonate
Investigations done to find the aetiology of
Increased risk of acquiring neonatal infection during
recurrent miscarriages
vaginal delivery
Karyotype of both partners
Give IG to neonate of a HBsAg +ve mother
Pelvic USS
Following are used in elective Caessarian section,
Maternal rubella status
i.v. atropine 0.6 mg
Sonoliysterography
Oral diazepam
Varicella zoster antibody status
Oral frusemide 50mg
In newborn to a diabetic mother
Sodium Citrate 30 ml
Hyperglycaemia
i.v. metochlopramide
Hypothermia
What of the following are tocolytics
Hypocalcemia
Dexamethasone
Jaundice
Nifedipine
Retinopathy
Indomethacin
Following drugs are harmful to be given during first
Verapamil
trimester of pregnancy
Terbutaline
Penicillin
Delayed puberty
Gentamicin
Commoner in girls than boys
Warfarin
Defined as > 11 years in girls
Thyroxine
Most common cause & constitutional delay in males
Sodium valproate

10

11

12

13

In girls it is an indication for karyotyping


Treatment with low dose testosterone induce secondary
sexual characteristics
Management of secondary haemorrhage following
Total abdominal hysterectomy
Broad spectrum antibiotics
High vaginal swab for culture and ABST
Norethisterone administration
Immediate laparotomy
Tight vaginal packing
Endometriosis give rise to
Amenorrhoea
Dysmenorrhoea outlasting duration of menses
Haemoptysis
Deep dyspareunia
Post menopausal bleeding
Genital ulcers caused by infective agents
Behets syndrome
Cervical Ca
Chancroid
Primary syphilis
Sarcoidosis
Regarding Herpes
Causes acute urine retention
Known to cause vulval CA
Is caused by RNA virus
Diagnosed by viral culture
Uterine Fibroids
Asymptomatic in majority
Main cause for dysmenorrhoea
Differentiated from adenomyosis by ultrasound
Hysterescopic excision is a management option
Contraindication for HRT
Pre induction cervical ripening in a post term
pregnancy is carried out by
ASOM
Extra amniotic sylpostrone infusion
Intracervical Foley
Intra vaginal NO releasing substance
Intra vaginal PGE2
Physiological changes in normal pregnancy
Reduction in BP in T2
Increase in pulse rate by 30%
Increase in RR by 50%
Reduction in Hb levels
Increase in renal blood flow
Management options in unruptured tubal pregnancy
Watchful expectations
Laporoscopic salphingectomy
Methotrexate
Oxytocin
Tranexaemic acid
Cause of IUGR includes
Down syndrome

A PROJECT OF MSWS AND MFSU- COLOMBO (2010/2011)

15

16

17

18

19

20

21

22

23

GDM
Chronic hypertension
Passive smoking
Polyhydramnios
Cu- IUCD
Causes dysmenorrhoea
Pearl index is 5
Contra indicated following dilatation and curettage
Safely inserted during menstruation
Recommended for 5 years
COCP
Increase ovarian CA
Increase endometrial CA
Increase risk of ectopic pregnancy
Contraindicated in uncomplicated DM
Increase cervical mucus
Seminal fluid analysis
Best form coitus interuptus
Indicated in secondary infertility
Abstinence for 2 weeks
Test done within 10 minutes
Tubal factor infertility
Chlamydiae
Mumps
Endometriosis
Vaginal candidiasis
Septic abortion
Diagnostic hysteroscopy
Helps to detect endometrial polyps
Used to exclude endometrial carcinoma
Uterine rupture is recognized complication
First line investigation in dysmenorrhoea
After the procedure 3 days hospital stay is mandatory
Relevant investigation in a 35 year old woman with
menorrhagia
Endometrial biopsy
Thyroid function test
Hb
Serum FSH / LH levels
Transvaginal scan
T/F
Reduced FSH found by Asherman syndrome
Reduced oestradiol suggest PCOD
Positive progesterone challenge test indicates presence
of oestrogen
Ovarian dermoid cyst
Cause torsion of the ovary
Cause urinary retention
Is treated by ultrasound guided aspiration
50% malignant
Had endometrial derivatives
In the normal menstrual cycle

5 | Page

a
b
c
d
24
a
b
c
d
e

LH surge occur on the 12th 15th days


Maximum progesterone is secreted on day 25
Oestrogen has a ve feedback effect on FSH
Ovulation occur with LH surge
CA cervix management
Chemotherapy adjuvant to radiotherapy
Hormone therapy adjuvant to radiotherapy
Pelvic exanguiation in early disease
Radical hysterectomy in early disease
Radiotherapy is used at all stages

OBSTETRICS AND GYNAECOLOGY


, ERPM-AUGUST 2010
1.
a.
b.
c.
d.
e.
2.
a.
b.
c.
d.
e.
3.
a.
b.
c.
d.
e.
4.
a.
b.
c.
d.
e.
5.
a.
b.
c.
d.

Ultrasound markers of down syndrome


Short femur
ASD
Increased nuchal fold thickness
Prominent nasal bone
Duodenal atresia
Treatment of genuine stress incontinence
Tension free vaginal banding
Pelvic floor exercise
Reduce fluid intake
Oxibutinin
Burch colposuspension
Abnormal features of CTG include
Baseline heart rate 100 beats per min
Baseline variability of 15 beats per min
Presence of late decelerations
Presence of variable decelerations
Sinusoidal pattern
Regarding labor with valvular heart disease
Ampicillin and gentamycin can be given prophylactically
In trenderlenberg position can be delivered
Can give epidural analgesia
Liberal use of 5% dextrose in dehydration
Routine assisted vaginal delivery (instrumental)
PPROM
Associate with polyhydramnios
Treated with Ciprofloxacin
Give steroids fortnightly
Causes chorioamnionitis
e.
Associated with severe preeclampsia
6. Composition of vaginal bacterial flora depends on
a.
IUCD usage
b.
Recent antibiotic use
c.
Stage of menstrual cycle
d.
Time of last sexual intercourse
e.
Use of HRT
7.
Causes for primary dysfunctional labour
a.
Breech presentation

b.
c.
d.
e.

Brow presentation
Fetal macrosomia
Fundal fibroid
Occipito posterior position
8.

a.

Following drugs and their side effects are


correctly paired
Methotrexate hair loss
b.
Ergometrine hypertensive encephalopathy
c.
Heparin fetal SDH
d.
Methyldopa fetal cleft palate
e.
Tamoxifen endometrial polyp
9.
Complications of vaginal hysterectomy
a.
Acute retention of urine
b.
Vault hematoma
c.
vault prolapse
d.
paralytic ileus
e.
Ureteric injury
10. Partial H.mole
a.
Has 69 chromosomes
b.
is best treated by abdominal hysterectomy
c.
Is locally invasive
d.
Is treated with radiotherapy
e.
Metastasis to the brain
11. Following disorders leads to subfertility
a.
Anorexia nervosa
b.
Diabetes insipidus
c.
Hypogonadotrophic hypogonadism
d.
PCOS
e.
Hyperprolactinaemia
12. Surgical treatment of UVP
a.
Abdominal hysterectomy
b.
Anterior colporrhaphy
c.
Manchester repair
d.
Sacrohysteropexy
e.
Vaginal hysterectomy
13. Regarding dermoid cyst
a.
Characteristically unique Doppler flow pattern
aiding detection
b.
Bilateral 105 cases
c.
Effective treatment with DMPA
d.
Common age group is 20-30 years
e.
Where rupture causes chemical peritonitis
14. Physiological changes during pregnancy
a.
Decreased fibrinogen level
b.
Increased cardiac output
c.
Increased blood supply to skin
d.
Increased intestinal motility
e.
Decreased blood pressure during 2nd trimester
15. Dilatation and curettage for post menopausal
bleeding
a.
Helps to establish the diagnosis of adenomyosis
b.
Is avoided if endometrial thickness is 2 cm
c.
Is inferior to hysteroscopically directed biopsy

d.
e.
16.
a.
b.
c.
d.
e.
17.
a.
b.
c.
d.
e.
18.
a.
b.
c.
d.
e.
19.
a.
b.
c.
d.
e.
20.
a.

Is used to confirm diagnosis of cervical carcinoma


Used to stage uterine carcinoma
Episiotomy
Midline episiotomy can cause rectovaginal fistula
Can cause genital tract prolapse
Is known to cause dyspareunia
Is not done for ventouse delivery
Is done when head crowning
Malignant potential seen in
PID
Nabothian cyst
Endometrial atypical hyperplasia
Cervical ectropion
Bartholian cyst
Regarding pelvic inflammatory disease
Causes Menorralgia
Can be caused by Chlamydia trachomatis
Can be diagnosed by trans vaginal ultrasound scan
Causes endometrial carcinoma
Causes chronic pelvic pain
Features of obstructed labour
Fetal distress
Fresh blood in the urine
Excess caput and moulding
Saggital suture in the transverse diameter
Edematous cervix
Regarding labour
Epidural analgesia does not slow 1 st stage of the
labour
b.
Flexion of the head increases the presenting
diameter
c.
Latent phase cervical dilatation is 1cm/hour
d.
Induction increased up to 5 contractions per 10
mins
21. Symptoms of adenomyosis
a.
Menorrhagia
b.
Dyspareunia
c.
Chronic pelvic pain
d.
Post coital bleeding
e.
Inter menstrual bleeding
22. In post menopausal women
a.
Persistent increase in LH leads to amenorrhea
b.
Absence of primordial follicle
c.
FSH levels comes to normal in 2-3 years
d.
Ovaries produce androgen
e.
If uterus intact, progesterone should included in
HRT
23. Face presentation
a.
Ventouse can be applied
b.
Can be delivered by vaginally if mento anterior
position
c. Maxilla and chin can be palpated
d. Is seen in 5% of labours

A PROJECT OF MSWS AND MFSU- COLOMBO (2010/2011)

e. Mento-posterior position can be rotated in mentoanterior by applying kjeland forceps


24. Conditions causing low Platlet count
a. Antiphospholipid syndrome
b. vWD
c. HELLP syndrome
d. Sever preeclampsia
e. ITP
25. HRT
a. Decrease the risk of colorectal carcinoma
b. Thrombo-embolism chance is decreased
c. Decreased the coronary artery disease
d. Increase the libido
e. Increase the risk of breast carcinoma
26. DMPA as a use of contraception
a. Causes amenorrhea
b. Causes cervical ectropion
c. Does not affect ovulation
d. Contraindicated in breast feeding
e. Given as monthly injection
27. Measurements of hormone levels in correct days
a. Day 14-FSH
b. Day 14-LH
c. Day 21-Oestrogen
d. Any day Prolactin
e. Any day Testosterone
28. Causes for preterm labour
a. Cervical incompetence
b. Multiparity
c. Preeclampsia
d. Previous myomectomy
e. Recurrent UTI
29. Absolute contraindications for ECV
a. History of APH in present pregnancy
b. History of classical Cesarean section
c. Negative blood group of the mother
d. Placenta previa
e. Primigravida
30. Absolute contraindications of combine oral
contraceptive pills include
a. Active liver disease
b. Iron deficiency anemia
c. Past history of eclampsia
d. Pulmonary hypertension
e. Presence of varicose veins
31. Maternal mortality reduced by
a. Periconceptional folic acid
b. Tetanus toxoid
c. Correction of anemia
d. LSCS done according to request
e. Increased maternal literacy
32. Measures adopted to intra uterine resuscitation
of fetus
a. Tocolytics

6 | Page

b. Amino infusion
c. Positioning of the mother
d. Oxygen administration
e. Hydration of the mother
33. Squamous cell carcinoma of the cervix
a. It has familial tendency
b. Effectively treated with HPV vaccine
c. Metastasis to the liver occur earlier than ovarian
carcinoma
d. Speculum examination used for down staging
e. Risk reduced by use of condoms
34. Defects are correctly matched
a. Bicornuate uterus- premature labour
b. Cervical incompetence- 1st trimester miscarriage
c. Mllerian agenesis secondary amenorrhea
d. Rudimentary uterine cornu ectopic pregnancy
e. Wolffian duct abnormality vaginal cyst
35. Advice during prepregnancy counseling to a
women who has recently had an unexplained
still birth includes
a. Delay the next pregnancy by at least 1 year
b. Have a glucose tolerance test done if not already
performed
c. Life style modification if the mother is obese
d. Take vitamins preconceptionally
e. The need to take peri conceptional folic acid

d)
4)

5)

6)

SURGERY
FACULTY OF MEDICINE- COLOMBO
2003 A/L 2009 DECEMBER
1)

2)

3)

30 year old man admitted with a history of blunt


trauma to left chest complaining of chest
pain.PR-94, BP-100/70, the possible injuries
include
a)
Flial chest
b)
Ruptured diaphragm
c)
Left sided pneumothorax
d)
Pulmonary contusion
e)
Ruptured spleen
Drugs that should be stopped before a major
surgery
a)
Clopidogrel
b)
Warfarin
c)
Prednisolone
d)
Losartan
e)
OCP
Regarding supracondylar # of humerus
a)
Common on fall on outstretched hand
b)
More common in children
c)
Can result in brachial artery damage

7)

8)

9)

e)
T/F
a)
b)

Commonly by open reduction and internal


fixation in children
Results in varus deformity of elbow

d)
10)

Osteosarcoma metastasis via blood spread


Osteoradionecrosis
is
a
recognized
complication of radiotherapy
c)
Acute haematogenous osteomyelitis is
commoner in children than in adults
d)
Treatment of Developmental dysplasia of hip
starts at 1st week of life
e)
Slipped upper femoral epiphysis is more
common among males
Risk factors for breast cancer includes
a)
Hx of breast CA on contralateral side of the
breast
b)
Family history
c)
Breast feeding
d)
Nulliparity
e)
OCP
Euthyroid patient with MNG was started on
Carbimazole 3 months back. That time she had
normal T3, T4. She was on aspirin and
Clopidogrel for 1 month for IHD. Suddenly she
has developed neck swelling and discomfort
over 48 hours which of the following are true
regarding her management
a)
Stop Carbimazole
b)
Stop aspirin and Clopidogrel
c)
Urgent neck CT scan
d)
Arrange immediate tracheostomy
e)
Do a core biopsy
A G5 year old male is presenting with a history of
abdominal pain, Back pain, abdominal distension
and reduced BP. Possible D/Ds includes
a)
Leaking AAA
b)
Renal colic
c)
Acute pancreatitis
d)
Typhoid perforation
e)
Myocardial infarction

a)
Commonly affect young males
b)
More common in heavy smokers
c)
Both upper and lower limbs are affected
d)
Veins are affected
e)
Proximal arteries are commonly affected than
distal arteries
21 year old operated for acute appendicitis. On
operation caecum and appendix appears normal.
Further management includes
a)
Should remove the appendix
b)
Look for skip lesions suggestive of Crohn's
disease
c)
Meckel's diverticulum should be looked for

11)

12)

13)

14)

15)

16)

17)

Right side hemicolectomy is an accepted


method of treatment
e)
Metronidazole is given for 7 days after surgery
Acute suppurative otitis media
a)
Commonest
cause
is
streptococcus
pneumonia
b)
Relieved by myringotomy
c)
Cause mastoiditis
d)
Known to cause subdural abscess
e)
Have to suspect in a child with fever and
irritability
These suture materials are absorbable
a)
Catgut
b)
Polypropelene
c)
Black silk
d)
Polyglactine
e)
Polyamide
Following affects the prognosis of a cancer
a)
Tumour size
b)
Duration of symptoms
c)
Spread to local LN
d)
Spread to adjacent organs
e)
Degree of differentiation
Cross sectional image can be seen
a)
Supine X-ray
b)
CT
c)
Trans rectal USS
d)
Trans abdominal USS
e)
Technetium99 scan for Meckels diverticulum

a)
Calculi are more frequent than parotid gland
calculi
b)
Associated with mumps infection
c)
Presents as a swelling during eating
d)
Calculi in duct is removed under local
anaesthesia
e)
Calculi in gland, sialadenectomy is an option
Anal fissures
a)
Majority heal spontaneously without operative
treatment
b)
Anterior anal margin is the commonest site
c)
2% Diltiazem paste is used in treatment
d)
Anal mucosal trauma is the commonest cause
e)
Internal
sphincterotomy
reduces
the
recurrence rate in chronic fissure.
Regarding acute scrotal pain
a)
Testicular torsion is unlikely in a person aged
>20 years
b)
Torsion appendix of epididymis is a cause.
T/F regarding haematuria
a)
Commonly due to UTI in children
b)
Renal Cell CA is a cause

A PROJECT OF MSWS AND MFSU- COLOMBO (2010/2011)

c)
18)

19)

20)

21)

22)

23)

First episode of frank haematuria in an adult


needs urgent cystoscopy
d)
Normal USS can exclude bladder malignancy
Congenital pyloric stenosis
a)
Diagnosed by USS
b)
Familial tendency
c)
Abdominal mass palpable
d)
Hereditary
e)
Metabolic acidosis
Regarding sialolithiasis
a)
If occurs in duct can remove under LA
b)
Occur after mumps in submandibular gland
c)
If occurs in gland, is treated with
sialadenectomy
d)
Mostly occur in submandibular gland relative
to parotid
e)
Pain increases with food Intake
Stomach carcinoma
a)
Chronic atrophic gastritis is n common cause
b)
H. pylori infection is a protective factor
c)
Mainstay of treatment is surgery
d)
If there is a palpable mass felt in epigastric
region the tumour is inoperable
e)
Prognosis is better than colonic carcinoma
60 year old alcoholic, severe generalized
abdominal pain, previous USS revealed no
gallstones, ill looking with generalized guarding
of the abdomen. Serum amylase - 1600u/ml,
blood glucose - 260, AST 300IU/I, Ca -7.2mg/dl,
BU 20mmol/l. What of the following are indicated
in his management?
a)
Perform an ABG
b)
Insert CVP line
c)
Arrange to admit to ICU
d)
ERCP for drainage of bile duct
e)
Broad spectrum antibiotics
50 year old male with history of colonic
adenocarcinoma presents with LOA of recent
onset. USS of abdomen reveals suspicion lesion
in the liver. In this patient,
a)
Normal CEA levels rule out the possibility of
2ndries
b)
CT abdomenneeded to assess the operability
c)
Peritoneal deposits better seen in CT than
laparoscopy
d)
PT-CT is a tool to look for extra hepatic
metastasis
e)
Removal of the hepatic metastasisimproves
the disease free survival
Femoral hernia
a)
Less chance of strangulation than inguinal
b)
Commonest in female
c)
Medial border is femoral vein
d)
Below and lateral to pubic tubercle

7 | Page

e)

Absence of cough impulse can exclude future


strangulation

a.

FACULTY OF MEDICINE- COLOMBO


2004 A/L 2010DECEMBER
1.
a.
b.
c.
d.
e.
2.
a.
b.
c.
d.
e.
3.
a.
b.
c.
d.
e.
4.
a.
b.
c.
d.
e.
5.

Regarding Abdominal aortic aneurysms


(AAA)
Screening is cost effective in reducing
morbidity
Is best evaluated by measuring the AP
diameter
Magnetic resonance angiography is the
gold standard in diagnosis
A family history of AAA is known in 1525% of patients
Peripheral occlusive arterial disease is a
known association
In a patient with 35% superficial thermal
burns
Urethral catheterization is indicated
Crystalloid are preferred in fluid
resuscitation
Fluid requirement is calculated from the
time the patient is admitted to the
emergency ward
TPN is preferred to enteral nutrition
Management of pain becomes easier as
compared to patient with deep burn
Acute iritis causes
Hypopyon
Pain
Photophobia
Raised eye pressure
Visual loss
The following is/are true of surgical
conditions of the hand
Mallet finger results from rupure of
extensor expansion at the distal
interphalangeal joint
Trigger finger is treated by steroid
injection
Causative organism of acute paronychia
is Staphylococcus aureus
Injury to ulnar nerve in the forearm
results in flexion deformity of the distal
interphalangeal joint
In infection of the thenar space the hand
is kept elevated as part of treatment
Obtaining an immediate CT scan of
head is mandatory in the following
instances

b.
c.
d.

e.
6.
a.
b.
c.
d.
e.
7.
a.
b.
c.
d.
e.
8.
a.
b.
c.
d.
e.
9.
a.

A 35y old male is fallen from height, he


is conscious and rational and have an
abration over the forehead, with no skull
fractures
A 32y old female following a RTA is
admitted with GCS of 7/13
A 6 year old child collides with a push
bike and has vomited thrice on the way
to the hospital
A 40y old male is admitted following a
motor vehicle accident with multiple
trauma, his BP is 60/40mmHg and is
tachycardic, he has abdominal fullness
with GCS 10/13
A known epileptic is admitted after a fall,
since admission he has developed a fit
The following imaging features on a xray abdomen, would favour the acute
small bowelobstruction
Dilated loops of bowel with valvulae
conniventes
Intra peritoneal air around bowel loops
Multiple central bowel loops of 4cm
diameter
Acrobilia
Absence of large bowel gas
The following is/are true statements
Rupture of achilies tendon results in
weakness in dorsiflexiing the foot
Injury to the medial meniscus is caused
by twisting injury of the knee ## bearing
Perthes disease of the femoral head is
seen in boys at 7-8 years of age
Treatment for club foot is commenced
within few days of birth
Osteochondromas are usually seen
around the knee joint
The following is/are true of salivary
glands
Calculi of the parotid glands are usually
radio opaque
Evidence of facial palsy indicates
malignancy in a parotid tumor
Turmors of the submandibular gland are
more likely to be malignant than parotid
gland
The submandibular gland is located at a
lower level than its ductal opening
Recurrent parotitis of childhood often
has no identifiable aetiology
The following statements are true of the
abdominal wall
Lower midline laparotomy incision

b.
c.
d.
e.
10.
a.
b.
c.
d.
e.
11.
a.
b.
c.
d.
e.
12.
a.
b.
c.
d.
e.
13.
a.
b.
c.
d.
e.
14.
a.

Double breasting of the rectus sheath is


an accepted procedure in periumbilical
hernia
Majority of epigastric hernias have a
peritoneal sac
Patent urachus is a cause of umbilical
discharge
Smoking predisposes to recurrence of
abdominal wall hernia
Squamous cell carcinoma
Of the prepuce is treated by amputation
of the penis
Is a recognized complication of bowens
disease
Of
the
oesophagus
spreads
submucosally
In general, is less sensitive to
radiotherapy than adenocarcinoma
Is the usual type seen in the oral cavity
Following is/are true statements
Endoscopic sclerotherapy is effective in
controlling bleeding from oesophageal
varices
Best mode of treatment of hepatocellular
carcinoma is surgery
Liver is the usual site of metastases in
prostate cancer
Choledochal cysts may involve both
intra and extra hepatic biliary system
Peri ampullary cancer has a better
prognosis than hilar cholangiocarcinoma
Following presentations should raise
the suspiction of splenic injury in blunt
force trauma
Contusion over the left lower chest
Haematemesis
Left shoulder tip pain
Obliteration of liver dullness
Severe pallor
Regarding fracture healing
Bone remodeling is a complete within 6
weeks of injury
Internal fixation accelerates bone
healing
Non-union may be caused by soft tissue
interposition
Woven bone is replaced by lamellar
bone
The amount of callus formation is
proportional to the mechanical stress at
the fracture site
The following statements are true
Constipation is a causative factor in
diverticular disease

A PROJECT OF MSWS AND MFSU- COLOMBO (2010/2011)

b.
c.
d.
15.
a.
b.
c.
d.
e.
16.
a.
b.
c.
d.
e.
17.
a.
b.
c.
d.
e.
18.
a.
b.
c.
d.
e.
19.

Fresh bleeding per rectum is a feature of


coecal cancer
Passage of foul smelling tar like motion
indicates malabsorption
#?? Angiodysplasia of the colon is
treated by
Acute otitis media
Can be associated with conductive
hearing loss
Can be due to nasopharyngeal
carcinoma
Is usually due to viral infection
Is treated by myringotomy
Occurs more commonly in children than
adults
Possible causes of stridor within 4
hours after total thyroidectomy for
thyroid malignancy include
Hypocalcaemia
Neuropraxia of recurrent laryngeal nerve
Pethidine over dose
Haematoma
Tracheomalasia
The following statements are true
regarding infections of the hand
Passive motion is painful in flexor
tendon sheath infections
Pulp space infection results in avascular
necrosis of the ten#
Incision and drainage is done under
local anesthesia
Early mobilization of figure is not
advicable
Swelling is more in the dorsum than the
palmar aspect
Which of the following statements are
true regarding congenital hypertrophic
pyloric stenosis
Increased appetits is a feature
Metabolic alkalosis is a complication
Is diagnosed with an abdominal
ultrasound scan
Projectile vomiting is seen from the first
day of birth
Gastro-jejunal bypass is the treatment of
choice
The following statements are true
regarding IV fluids
a.
Insensible losses replaced with
5% dextrose
b.
1000ml of Hartmann supply
adequate K+ for adult male per
day
c.
Normal saline supplies energy

8 | Page

d.

20.
a.
b.
c.
d.
e.
21.
a.
b.
c.
d.
e.
22.
a.
b.
c.
d.
e.
23.

a.
b.
c.
d.
e.
24.

1000ml of normal saline supplies


daily Na+ required for male
e.
Colloid preferable for crystalloids
to rapidly replace intravenous
deficit
In a patient with hyperthyroidism and
goiter, surgery
is preferred over
radioactive iodine, if
The patient is 40 years old and having a
small diffuse goiter (WHO grade 1)
The patient is 25 years old with a large
multinodular goiter (WHO grade 4)
There is a solitary thyroid nodule which
is hot on radio isotope scan
There is a multinodular goiter with
retrosternal extension
There is follicular proliferation on FNAC
WOTF are T/F regarding ulcers
Majority of the patients with rodent
ulcers have palpable regional lymph
nodes
When long standing, should be treated
with systemic antibiotics
High ligation is contraindicated in the
presence of a venous ulcer
Those with sloping edges are
characteristic of healing ulcers
Malignancy occurring in a burn scar is
termed Majollins ulcer
In a 50 year old otherwise healthy
patient
with
billiary
calculi,
cholecystectomy is indicated if there is
A history of biliary colic
A history of flatulence related to meals
Hyperlipidaemia
Obstructive jaundice due to CBD calculi
Porcelain gallbladder
A 25 year old girl is complaining of
severe pain on defaecation of 3 days
duration ## has also moticed blood
streaks on the stols, with regards to this
patient
Urgent proctoscopy is indicated
Ulcerative proctitis is a likely diagnosis in
the presence of stools of normal
consistency.
If a fissure in ano is seen on
examination, laxatives should be
avoided/
Carcinoma of the rectum needs to be
excluded
Glyceryl trinitrate ointment is indicated in
the presence of a fissure
The following statements are true

a.
b.
c.
d.
e.
25.
a.
b.
c.
d.
e.
26.
a.
b.
c.
d.
e.
27.
a.
b.
c.
d.
e.
28.
a.
b.
c.
d.
e.
29.
a.
b.

Pulse oximetry measures the arterial


coxigen tension
Capnography measures expired carbon
dioxide
ECG monitoring is useful to detect the
cardiac output
Non invasive blood pressure readings
are accurate in a patient with
hypovolemic shock
A probe placed in the oesophagues
measures core temperature
Emphysema thoracis
Is a complication of haemothorax
Is drained by a size 18 intercostal tube
in an adult
Leads to pleural mesothelioma
Is characterized by formation of a thick
cortex of fibrin
If chronic, is treated by rib resection
A 50 year old woman is diagnosed with
ileo-femoral deep vein thrombosis #, an
anterior resection in this patient,
Early mobilization is associated with a
greater risk of pulmonary embolism
Treatment is initiated with heparin
A caval filter is not recommended at this
stage
Warfarin therapy should be continued for
up to six months
Long term aspirin is recommended
Paralytic ileus
Is caused by handling of bowel during
surgery
Is a manifestation of hypokalaemia
Occurs in acute pancreatitis
Improves with regular high saline
enemata
Is managed with nasogastric suction and
intravenous fluids until peristalsis returns
The following disease and procedures
##
Anal fissure application of a seton
Gastric fundal varices endoscopic
band ligation
Calculus at the vesico-ureteric junction
ESWL
BPH - TURP
Papillary carcinoma of the thyroid total
thyroidectomy
SUFE
Has a peak incidence between 5 and 10
years
Is often unilateral

c.
d.
e.
30.
a.
b.
c.
d.
e.
31.
a.
b.
c.
d.
e.
32.
a.
b.
c.
d.
e.
33.
a.
b.
c.
d.
34.
a.
b.
c.
d.
e.
35.
a.
b.
c.
d.

Is a recognized cause of AVN of the


femoral head
Is best diagnosed by USS of the hip
Require internal fixation
Regarding breast diseases
Mastalgia
is
commoner
in
premenopausal women
Fibroadenomata are derived from the
breast lobule
Lactational breast abscess are caused
by lactobacillus
Duct ectasia results in nipple discharge
Atypical hyperplasia is associated with
an increased risk of breast cancer
Regarding undescended testis
The risk of malignancy is reduced after
orchidopexy
Should be operated before the first
birthday
Predisposes to orchitis
Predisposes to torsion
Is associated with inguinal hernia
The following incisions are appropriate
in removal of the named organ
Loin incision kidney
Lanz incision appendix
Combined cervical and upper midline
abdominal incisions oesophagus
Inguinal incision seminoma of testis
A roof top (bilateral subcostal) incision
the head of pancreas.
Regarding day surgery
Inguinal hernia repair is a suitable
procedure
Is not suitable for conditions requiring
general anesthesia
Is not suitable for children
Should not be undertaken for patients
with co-morbid factors
Surgical site infection can be reduced
by
Hand washing
Prophylactic antibiotics
Peri-operative warming of the patient
Minimizing blood transfusion
Using topical antibiotics
Regarding pelvic abscess
Laparotomy is the preferred treatment
Is a known complication of acute
appendicitis
Causes frequent passage of small,
mucioid stools
Is known to cause intestinal obstruction

A PROJECT OF MSWS AND MFSU- COLOMBO (2010/2011)

e.
36.
a.
b.
c.
d.
e.
37.
a.
b.
c.
d.
e.
38.

a.
b.
c.
d.
e.
39.
a.
b.
c.
d.
e.

Is unlikely if abdominal palpation is


normal
Regarding achalasia cardia
24 hour monitorin of oesophageal pH is
indicated
rat tail appearance on barium swallow
is characteristic
Dysphagia is more for liquids than for
solids
Diagnosis is confirmed by oesophageal
manometry
Is treated by cardiomyotomy
Regarding urethral strictures
Anterior strictures are commonly caused
by instrumentation
watering can perineum is a
complication
Falling astride usually leads to prostatic
urethral stricture
Urethrography is used for diagnosis
Optical urethrotomy is a treatment option
a 25 year old man has undergone an
exploratory laporatomy for liver
laceration and he develops fever on the
10th post operative day, likely causative
include
malignant hyperpyrexia
abscess in the right lobe of liver
UTI
Intraperitoneal abscess
Metabolic response to trauma
T/F regarding local anesthetic agents
Bupivacaine is used in spinal anesthesia
Duration of action of lignocaine is longer
than that of bupivacaine
Convulsions are a feature of local
anesthetic toxicity
Addition of adrenaline to lignocaine wil
increase its duration of action
Maximum safe dose of lignocaine
without adrenaline is 3mg/kg

PEDIATRICS
FACULTY OF MEDICINE- COLOMBO
2003 A/L 2009 DECEMBER
1)
a)
b)
c)
d)
2)
a)

Autosomal dominant inheritance


Haemophelia
Tuberous sclerosis
Thalassemia
Vitamin D resistant rickets
Complications of Staphylococcus infections
Lung abscess

9 | Page

b)
c)
d)
e)
3)
a)

Empyema
Pneumothorax
Micrometastasis
Septic arthritis
T/F
Presence of splenomegaly excludes Mycoplasma
infection
b) Respiratory rate of 40 is abnormal in newborn
c) Bronchiectasis is a complication of Pertussis
d) CXR pattern in IRDS - areas of consolidation and
hyperinflation
e) X-ray features of Bronchiolitis is similar to X-ray of
asthma
4) Regarding childhood malignancies
a) Retinobtasloma with optic nerve involvement is not
treated with annucleation
b) Infratentorial more common than supratentorial
c) Neuroblastoma present as intrathoracic tumour
d) ALL - bad prognosis if present between 2-4 years
e) AML - commoner in Down syndrome
5) Hypokalemia occurs in
a) Loop diuretic therapy
b) Hypertrophic pyloric stenosis
c) Distal renal tubular acidosis
d) Gastroenteritis
e) Treatment with Spironolactone
6) Scabies in an infant treated with
a) Permethrin
b) Gamma BHT
c) Benzyl benzoate
d) Sulfur ointment
e) 1% hydrocortisone
7) Following are commonly matched
a) Deja vu - temporal lobe epilepsy
b) Night terrors - petit mal
c) Vacant episodes - Abscense seizures
d) Aura of fear - Benign rolandic epilepsy
e) Development regression - West syndrome
8) In Iron deficiency anemia
a) Increased red cell distribution width
b) Increased red blood cell count
c) Reduced mean corpuscular volume
d) Reduced MCHC
e) Increased MCH
9) Short stature is a recognized finding in
a) Klinefelter syndrome
b) Marfan syndrome
c) Achondroplasia
d) Hypothyroidism
e) Emotional deprivation
10) In Kwashiorkor
a) Oedema is due to heart failure
b) Has high mortality than marasmus
c) Have hypothermia

d) Appetite good
e) When giving nutritional treatment, hospital admission is
essential.
11) T/F
a) Erythromycin - Stevens-Johnson syndrome
b) Chloramphenicol - Intra vascular haemolysis
c) Carbamazepine - Hirsutism
d) Vit. K - Increased liver enzymes
e) Cyclophosphamide - increased risk of malignancy
12) In Vit. A deficiency
a) Usually subclinical
b) Bitot spots are reversible
c) Associated with respiratory infection
d) Serum ferritin is low
e) Commonly cause night blindness
13) Regarding congenital hypothyroidism
a) Early closure of anterior fontanel is seen
b) Thyroid dysgenesis is the main cause
c) Majority asymptomatic at birth
d) Neonatal screening is useful in diagnosis
e) Persistent neonatal jaundice is a recognized feature
14) Rubella infection causes
a) Orchitis
b) Thrombocytopenic purpura
c) Arthritis
d) Lymphadenopathy
e) Cataract
15) Cardiac syncope caused by
a) AS
b) Mitral incompetence
c) PS
d) HOCM
e) Congestive cardiac failure
16) X-ray findings in cardiac diseases
a) Boot shaped heart in TOF
b) Egg on side in tricuspid atresia
c) Figure8 in transposition of great arteries
d) Pulmonary plethora in VSD
e) Globular heart in pleural effusion
17) Loud 2nd heart sound in
a) Uncomplicated VSD
b) TOF
c) Coarctation of aorta
d) PDA with Eisenmenger syndrome
e) Transposition of great vessels
18) The following are true regarding constipation in
childhood
a) Soiling is a feature
b) Palpable abdominal faecal masses is an indication for
enema
c) Haemorrhoids is a feature
d) Most have a GI pathology

e) Need long term treatment


19) Regarding Post streptococcal glomerulonephritis
a) Monthly follow up with benzathine penicillin
b) Need to monitor for hypertension
c) Elevated ASOT is seen
d) Common among preschool children
e) Diuretic phase is there in recovery
20) Regarding hypertension in AGN
a) Associated with bradycardia
b) Presents with frontal headache
c) Leads to papilledema
d) Treated with Nifedipine
e) Commonly persists beyond childhood
21) Complications following nephritic syndrome
a) Abdominal pain
b) Increased urine output
c) Pulse pressure of 20mmHg
d) Frothy urine
e) Cushinoid facies
22) T/F
a) Early detection of hearing impairment improves school
performances
b) Ishihara test is used to assess the colour vision
c) Visual acuity cannot be tested until child can read letters
d) Draw man test use to assess the cognition
e) Correction of prematurity done at 3 years
23) Regarding CHDR
a) Gender specific
b) Up to 5 years only
c) Include information on childhood feeding
d) Has information on siblings
e) Has information on pubertal development
24) Development delay seen when
a) Not walking by 15 months
b) Not talking meaningful 3 words at 9 months
c) Not self feed by cup at 9 months
d) Stranger anxiety at 9 months
e) No mature pincer grasp at 6 months

PAEDIATRICS
2004 A/L- DEC 2010 COLOMBO
1) Complimentary feeding,
a Commencing before 17 weeks leads to increase allergy
in childhood
b Egg yolk is not introduced till 9months
c Smoothly smashed food should be given till 1 year
d Fat restriction is mandatory in obese infants
e Adult diet by 1 year
2 Febrile convulsion,
a Occur after 48 hours of fever
b Treatment with long term anti-convulsants
c Neuro imaging is not needed

A PROJECT OF MSWS AND MFSU- COLOMBO (2010/2011)

d DPT is contraindicated
3 Acute flaccid paralysis,
a Polio myelitis
4
a
b
c
d
e
5
a
b
c
d
e
6
a
b
c
d
e
7
a
b
c
d
e
8
a
b
c
d
e
9
a
b
c
d
e
10
a
b
c
d
11
a
b
c
d
e
12
a

Past exposure vaccination is effective in preventing,


Chicken pox
Tetanus
Rabies
Polio
JE
Brain tumour in childhood,
Majority are metastasis of primary tumours
Personality changed
Resistant epilepsy
ICP increased in supra-tentorial more than infra-tentorial
Short stature is a long term complication of treatment
Constipation in childhood,
Rectum is empty
Lead to faecal incontinence
Cause for reduced anal sphincter tone
Treated with poly ethelene glycol
Causes toxic megacolon
18 month old baby is able to,
Kick a ball
Drink from a straw
Pick up dirt and put in mouth
Dry by night
Speak in two word sentences
Skin lesions that only require reassurance
Erythema marginatum
Milia
Urticaria
Toxic epidermal necrolysis
Erythema toxicum
Cytogenetics in Down syndrome,
Non dysjunction is a feature in young mother
Worse prognosis if mosaicism
Less risk if father is the carrier
t14:21 is not compatible with life
non dysjunction risk increases with maternal age
Dyslexia,
Commoner in female
Associated with tics
Commonly presents with reading difficulty
Intelligence is below average
Regarding mental retardation,
80% IQ 50-70
Commonest chromosomal abnormality is Down
syndrome
Diagnosed by MMSE
Associated with low social class
Distress shown by behaviour than words
Features suggesting of child abuse,
Compound fracture of Humerus

10 | P a g e

b
c
d
e
13
a
b
c
d
e
14
6.
7.
8.
9.

Sub dural haemorrhage


Multiple bruising of same age
Fundal haemorrhage
Recurrent vomiting
Regarding neonatal resuscitation,
Head is kept in neutral position
Ventilation commence with inflation breaths
Jaw thrust is indicated when chest expansion is absent
Digoxin is given when heart rate <60 b.p.m.
Majority need endo-tracheal intubation
Leptospirosis,
Severe myalgia is a early feature
Microscopic haematuria is seen
Prothombin time is increased
Myocarditis lead to circulatory
collapse
10.
Benzathine penicillin is the drug of
choice
15 Subsequent dose of DPT is not given if
a Severe local reaction to previous dose of DPT vaccine
b Prolonged screening within 1st 24 hrs of the last dose
c Simple febrile convulsion
d Developmental regression
e Anaphylaxis reaction following previous vaccination
16 Rheumatic carditis
a Pan carditis
b Congestive cardiac failure
c Is uncommon if RF occur in preschool age
d Causes rapid sleeping pulse
e Corticosteroid is 1st line
17 PUV
a Exclusively in males
b Diagnosed by MCUG
c Associated with VUR
d Surgically treated prevents CRF
18 Cause for painless haematuria
a Cystitis
b Intra vascular haemolysis
c AGN
d Ig A nephropathy
e SLE
19 Innocent murmur,
a Systolic
b Radiates
c Intensity varies with posture
d Has rhythmic abnormality in ECG
e Disappears with exercise
20 Causes for biventricular hypertrophy,
a PDA
b TOF
c Small VSD
d OS ASD
e Pulmonary stenosis
21 Regarding childhood pneumonia,

a Lobar pneumonia most commonly caused by viral


infection
b Atypical organisms best treated with 2nd generation
cephalosporins
22 Bronchiolitis,
a Segmental atelectasis in CXR
b Humidified oxygen increases insensible losses
c Treated effectively with cefotaxime
23 Clinical features of retropharyngeal abscess,
a Hoarseness
b Epistaxis
c Torticollis
d Retraction of the neck
e Widening of retropharyngeal space
36. Macrolides,
a Bactericidal
b First line drug in community acquired atypical
pneumonia
c Delay gastric emptying
d Cholestatic hepatitis
e Causes severe arrhythmias when given with artemazole
37. Activated charcoal,
a Useful in phenobarbitone poisoning
b Useful in iron overdose
c Toxic if aspirated to lungs
d Acts by detoxifying the poison
e Not useful in a child with acute paracetamol poisoning 2
hours ago
38. Diaphragmatic hernia,
a Systolic murmur heard over the precordium
b Scaphoid abdomen
c Absent breath sounds on one side of the chest
d Mediastinal shift to one side
e Double bubble appearance

PAEDIATRICS AUGUST-2010 ERPM


Minimal change Nephrotic syndrome
Pneumococcal vaccine is indicated
Renal biopsy if steroids resistant
Definition of frequent relapsing 4 per year
No steroid dependency is minimal change disease
FFP is better than CPP in treatment
Epiglottis
Hib is the comment organism
Horse voice
Oropharyngeal causes drooling of saliva
Treated with IV Cefotaxime
Diagnosed by examining throat
Asystole
Commonest causes of cardiac arrest is children
CPR 15:2 compressions: breaths

Adrenalin 1:1000 given


Indications for defibrillation
Occasional p waves cannot exclude diagnosis
Primary nocturnal enuresis
Delay in development of nerves is the cause
Mild cases can be managed by a general practitioner
Bell and pad is the commonest method used in sri lanka
Treatment with TCAs will lead to complete recovery
Diagnosed at 3 years when wet at night
Causes of constipation
Phototherapy
Hirschprungs disease
Pb poisoning
Formula feeds
Cerebral palsy
Innocent murmur
Left sternal angle
Change with position
Systolic
Thrill
Heave
Mid upper arm circumference
No specific age for measurement
The midpoint between the acromion and the oleocranon
Better for mass screening
Up to 10 years
Not a measure of obesity
ADHD
Equal sex ratio
Age less than 3 years
Treatment with methylphenidate
Symptoms are seen only at home
School performance is usually good
Match the following with each other
Autism stereotyping
ADHD hair plucking
Conduct disorder theft
Tourettes syndrome- vocal tics
Anorexia nervosa amenorrhea
EPI
BCG vaccine has >90% coverage
DPT and Heb B at 18 months
aTD at 10 years
Rubella at 13-14 years to females
OPV at 18 months
True or false regarding
weight gain of 25-30g/day during the 1st 6 months
Up to 10% weight loss in the 1st week is normal
CHDR based on multi centered date
Birth weight corresponds to mid parental height
Length is measured up to 2 years
Development
Pincer graft at 6 months
Riding a tricycle at 3 years

A PROJECT OF MSWS AND MFSU- COLOMBO (2010/2011)

Double syllable babbling by 6 months


Head control by 3 months
Sits with support by 3 months
Hemophilia A with joint bleeding
APTT prolonged
PT prolonged
BT normal
CT normal
Platelets are low
Causes of congestive cardiac failure in infants
SVT
Complete heart block
Dilated cardiomyopathy
Pulmonary stenosis
VSD
Immunization in EPI
JE given subcutaneously at 1 year
DTP is contraindicated in HIV
JE is given at 9 months
MR at 3 years
JE is a live vaccine
Recurrent pneumonia
H-type trachea-esophageal fistula
IgA deficiency
Tetralogy of fallot
Primary ciliary dyskinasia
GORD
Cerebral palsy
Due to a non-progressive lesion
Lower limbs more affected in diaplegic cerebral palsy
Complicated by GORD
Prone to seizures
Choreoathetoid CP kernicterus is the cause
Reduced or absent tendon reflexes
Cerebral palsy
Spinal Muscular Atrophy 1
Duchene Muscular Dystrophy
Friedreichs ataxia
Guillian barre syndrome
Genetic disorders
Congenital adrenal hyperplasia autosomal recessive
Hypophosphateneamic rickets Autosomal dominant
50% offspring will be carriers in AR carrier parents
X linked recessive does not affect females
Autosomal dominant skip a generation
Chronic diarrhoea
Secondary lactose intolerance
Rota virus
Immune deficiency
Giardiasis
Inflammatory bowel disease
Neonatal meningitis
High pitched cry

11 | P a g e

b.
c.
d.
e.
22.
a.
b.
c.
d.
e.
23.
a.
b.
c.
d.
e.
24.
a.
b.
c.
d.
e.
25.
a.
b.
c.
d.
e.
26.
a.
b.
c.
d.
e.
27.
a.
b.
c.
d.
e.
28.
a.
b.
c.
d.
e.
29.
a.
b.
c.
d.
e.
30.

Brudzinski sign commonly positive


Subdural effusion is a complication
Rifampicin prophylaxis in pneumococcal
Commonest cause is Group A streptococci
Mycoplasma
Common respiratory organism
Causes heamolytic anemia
Treated with amoxicillin
Causes meningoencephalitis
Rare in children
Post streptococcal Glomerulo nephritis
Commoner in pre-school age
Presents with massive edema
Treatment with frusemide is mandatory
Salt restriction in early management
Microscopic hematuria might persists for 1 year
Rheumatic fever
Type 2 hypersensitivity reaction
Chorea needs lifelong treatment
Chorea can occur with normal ESR
Small joint involvement is rare
Commonest valve involved is the mitral valve
Billiary atresia
HIDA scan is indicated
Acholic stools
Urobillinogen in the urine is high
Causes portal hypertension
Kasai procedure is a option of management
Lung function test
Increased reserve volume in asthma
Total Lung Capacity increases in asthma
FEV1/FVC reduced in asthma
Tidal volume reduced in GBS
Lung compliance increases with IRDS
Gestational DM
Hyaline membrane disease
Seizures
Hypoglycemia
Hypocalcemia
Hypothermia
Birth asphyxia causes
DIC
Intraventricular bleeding
Hyperglycemia
Seizures
Acute renal failure
Kerosene poisoning
Pinpoint pupils
Hyperventilation occurs due to stimulation of the
respiratory center
Convulsions is an early sign
Do not induce vomiting
Chest X-ray shows patchy shadows
In sleep

Sleep walking is recollected


Night terrors REM sleep
Night terrors Fully aware when awake
Night mare seizure disorder
Sleep walking confronting the child and gently
redirecting him or her to bed
Measles
Macular popular rash
Rash ulcerates
Incubation period of 14-21 days
Is effectively prevented when vaccinated at 9 months
Morbidity is high with pneumonia
Non-accidental injury
Multiple posterior rib fractures
Spleen / liver injuries
Spiral fracture in a 9 month old
Retinal hemorrhages
Depressed skull fracture
Coarctation of aorta
BP is high in legs
Absent femoral pulses
RVH in older children
Murmur between the shoulder blades
Associated with Turners
Drug treatment in children
Thyroxine before breakfast
Soluble insulin as intradermal
Pneumocystis carnii prophylaxis with co-trimoxazole
Hypertonic saline better than isotonic saline in dengue
shock
Ciprofloxacin is the drug of choice for typhoid
VACTERL
Upper GI obstruction
Cardiac anomalies
Renal anomalies
Vertebral anomalies
External ear anomalies
ALL
T-cell type is common
Presence of mediastinal mass
Can present with monarthritis
>50,000 WBC bad prognosis
CNS involvement indicated bad prognosis
In Sri Lanka
Maternity leave with pay 84 days
Special classes for disabled children in government
schools
500/= per month for thalasseaemic children
The compulsory age for schooling is 12 years
Guillen barre
Is characteristically an ascending paralysis
Causes permanent brain damage
IV immunoglobulin is the treatment option
Reflexes are exaggerated

Bladder and bowel incontinence is a common problem


Dengue
DIC is a complication
Colloids preferred to crystalloids in shock
Not seen in the 1st 6 months of life
Has 4 serotypes
Leakage phase occurs in the peak of the fever
In fallot Tetralogy
Cyanosis is present from birth
Heart failure occurs during 1st 6 weeks of life
Pansystolic murmur due to the VSD
Subclavian artery to pulmonary artery shunt will abolish
in cyanotic spells
Cerebral abscess is a known complication

PSYCHIATRY
FACULTY OF MEDICINE-COLOMBO
2003 A/L- 2010 DECEMBER
1)
a)
b)
c)
d)
e)
2)
a)
b)
c)
d)
e)

In Panic disorders,
Presents as recurrent chest pain
Commonly associated with cardiac arrhythmias
Effectively treated with Imipramine
Can be caused by Hypothyroidism
Significantly associated with Agarophobia
Recognized complications of Risperidone
Hyperprolactinaemia
Weight gain
Tardive dyskinesia
Akathesia
Precipitation of mania

3)
a)
b)
c)

Dissociative disorders
Acute on onset
Neurological symptoms rare
Patient consciously produces symptoms which are
beneficial
Fluoxetine is used to treat
High suicidal intent in PCM poisoning
Has collected tablets over the preceding week
Swallowing tablets in front of family members following
an argument
Seeking help after the act
Giving personnel possessions to a close friend the act
Denying the suicidal intent

d)
4)
a)
b)

c)
d)
e)
f)
5) Obsessional thoughts
a) Present in normal people
b) Best treated by reassuring patient that these thoughts
are normal
c) Best treated by Imipramine

A PROJECT OF MSWS AND MFSU- COLOMBO (2010/2011)

d)
e)
6)
a)
b)
c)
d)
e)
7)
a)

Are form of thought insertion


Can lead to suicide even in the absence of depression
Poor diagnostic patient interviewer includes
Taking notes during interview
Allowing the patient to cry
Closed questions
Allow relative to be present
Avoiding silence
T/F
Thought stopping in treatment of patient with specific
phobia
b) Analyzing dream is a key component of cognitive
treatment
c) Group therapy is effective in management of acute
symptoms of psychosis
8) Following drugs are effective as mood stabilizers in
bipolar affective disorder
a) Olanzapine
b) Lamotrigine
c) Carbamazepine
d) Lithium carbonate
e) Quetiapine
9) Regarding ECT
a) Done under GA
b) Needs muscle relaxation
c) Needs intubation
d) Needs to be connected to ventilation
e) Apply a local anaesthetic to the pads
10) Following are suggestive of Autism
a) Wide range of play activities
b) Poor language development
c) Overactive when usual environment is changed
d) Rejects when parents hug him
e) Has good eye contact
11) OCD in children can present with
a) Excessive hand washing
b) Impulsive behaviour
c) Slowing of school work
d) Stealing
e) Repeated movements
12) Normal people have
a) Ideas of reference
b) Auditory hallucinations
c) Irrational fear to harmless objects
13) WOF suggest ADHD
a) Doesn't listen when spoken
b) Lose pen and pencil at school
c) Doesn't hang out with peers
d) Repetitive movements
14) Abnormal grief reaction is more likely if
a) Dying is prolonged
b) Survivor is dependent on deceased
c) Survivor cannot express emotions

12 | P a g e

d) Survivor is having previous psychiatric illness


e) Deceased is an elderly
15) A 30 year old accountant complains that he is
unable to complete his work on time because he
is not comfortable until he checks figures at least
3 times
a) This condition is compatible with early dementia
b) This is likely precursor of a conversion disorder
c) Imipramine will be effective for the patient
d) Behavior therapy is likely to be helpful
e) Lithium prophylaxis is useful
16) In a 30 year old girl, who is unable to read and write
following are useful in making a diagnosis
a) History of birth asphyxia
b) Family history of reading and writing difficulty
c) Multiple somatic complains
d) Refused to talk with teachers and peers in the school
e) Episodes of febrile convulsion at the age of 2 years
17) Following are correct primary defense mechanisms
a) Regression is blaming someone else for one's own
mistake
b) Displacement is going into an earlier developmental
stage
c) Denial is implied when a person shows ignorance
d) Reaction formation
e) Entirely pathological
18) Regarding mental state examination
a) Horizontal furrows with wide palpebral fissures seen in
depression
b) Vertical furrows downward displacement of the corner of
the mouth seen in anxiety.
c) Bright colour wear- hypomania

ESSAY

MEDICINE
Faculty of Medicine
Colombo
December 2008

extensive anterior ST elevation myocardial infarction 2


months back. On examination his blood pressure was
210/110 mmHg. Heart rate was 110 bpm & random
blood sugar was 380 mg/dl. He was unable to walk or
stand. His ECG showed ST elevation & Q waves from
leads V1 to V5.
1.1List the anatomical sites of lesions that could explain
his neurological deficit. (05)
1.2Discuss the pathogenic mechanisms involved in his
neurologic deficit. (40)
1.3Outline the acute management of this patient in the
1st 24 hours after admission. (40)
1.4Outline the various aspects of long term
management of this patient. (15 marks)
2) Describe the treatment regimens you would prescribe
in the following situations, giving justification for each.
2.1 The empirical antibiotic regimen in an adult patient
with community acquired acute pyogenic meningitis.
(40)
2.2 The drug treatment of a 65 year old man with DM
with a BMI of 28kg/m2 & a glycosylated Hb of 8.5%.
(30)
2.3 The drug treatment of a young non-pregnant woman
who presented in thyrotoxicosis due to Graves disease.
(30)
3) Describe the following:
3.1 The management of a school boy bitten by a stray
dog. (40)
3.2 The management of a student nurse who has had
an accidental needle stick injury while drawing blood
from a patient being investigated for jaundice. (30)
3.3 The management of a patient with community
acquired pneumonia. (30)
4) Discuss the significance & the management of the
findings listed below.
4.1 The presence of microalbuminuria in a 40 year old
man with diabetes. (40)
4.2 The persistence of proteinuria in a 20 year old with
minimal change type nephritic syndrome after treatment
with steroids. (30)
4.3 The development of renal artery bruit in a patient
with resistant hypertension. (30)
5) A 16 years old boy presented with gum bleeding &
haematuria of 10 days duration
5.1 List 3 possible causes for this condition. (10)
5.2 Describe the features in the history & examination
that would help to differentiate between these 3 causes.
(50)

5.3 Indicate the laboratory investigations you will


request on this patient, explaining how they will help you
to arrive at a diagnosis. (40)

OBSTETRICS AND
GYNAECOLOGY

CLINICAL SCIENCES STREAM


2010 DECEMBER
MEDICINE PAPER 01
Q1. A 16 year old boy was admitted with a history of
fever for five days. He had noticed that his urine was
dark. He right hypochondria discomfort & nausea. There
was history of travel to Kataragama 2 weeks back on
pilgrimage.
On examination he had a few lymph nodes in the neck,
of less than 1cm in size. He was jaundiced &n there was
tender hepatomegaly (liver edge was felt 3cm below the
costal margin)
State two likely diagnoses. (10 marks)
1.1 State two clinical features (symptoms or signs)
that would support each of two diagnoses
mentioned in 1.1 (20 marks)
1.2State three laboratory investigations you would
do to differentiate between the diagnoses
mentioned, & describe the expected findings (30
marks)
1.3Describe the principles of treatment of the acute
condition & one of its complications in any one of
that you have mentioned in 1.1 (20 marks)
1.4Give the principles of controlling an epidemic of
one of the diagnoses mentioned in 1.1 (20 marks)
Q2. A 60year old lorry driver with a 5 year history of
shortness of breath & wheezing was admitted with
worsening breathlessness. He complained of cough
within yellow green sputum. He had smoked 35
cigarettes per day for the last 40 years.
On examination he was febrile & had a barrel shaped
chest. The accessory muscles of respiration were used
with the intercostals muscles in-drawing respiration. On
auscultation he had bilateral bronchi & scattered coarse
crepitations in both lungs.
2.1. State the most likely COMPLETE clinical
diagnosis (10 marks)
2.2. List three investigations you would do to confirm
your diagnosis, giving the expected findings
in
each case (30 marks)
2.3. Discuss the management of this patient (50
marks)
2.4. Name two possible complications he may
develop (10 marks)

UNIVERSITY OF COLOMBO, SRILANKA


FACULTY OF MEDICINE
END OF COURSE EXAMINATION OF THE CLINICAL
SCIENCES STREAM
(REPEAT) DECEMBER 2008
OBSTETRICS AND GYNAECOLOGY-1
1.
2.
2.1
2.2
2.3

Part A
A primigravida who was in labour found to be having
cervical dilatation of 6cm for 8 hours. Describe the
management of this patient up to the delivery. (100)
List five physiological changes that take place during
normal pregnancy. (15)
List five aggravating factors for cardiac failure during
antenatal period in a woman with mitral stenosis. (20)
List four antenatal cardiovascular complications of
moderate mitral stenosis. (20)
2.4 Outline the assessment of a woman at 38 weeks
of gestation with mitral stenosis. (20)
2.5 Outline the management of labour in a woman with
a mitral stenosis. (25)
3.

3.1
3.2
3.3

Part B
A 65 years old mother of two children has
presented with postmenopausal bleeding per
vagina. Her general, cardiovascular, respiratory
and abdominal examination did not reveal any
abnormality following a diagnostic D&C
histological examination of endometrial tissue
revealed a well differentiated adenocarcinoma of
the endometrium.
Briefly describe the management of this patient
(40)
Outline how you would follow up this patient (40)
Give five prognostic factors for her condition (20)

4.

A 38 year old mother is admitted to the antenatal


ward with reduced fetal movements for 12 hrs at
37 weeks of gestation.
4.1 Mention the important information you would
elicit in the antenatal history to arrive at a
possible cause for this presentation. (20)
4.2 Name two important investigations you would
perform on this patient giving reasons. (20)
Explain how you would manage this condition until
delivery. (60)

1) A 65 years old diabetic presented with sudden onset


weakness in left upper & lower limbs. He had an
A PROJECT OF MSWS AND MFSU- COLOMBO (2010/2011)

13 | P a g e

END OF COURSE EXAMINATION OF THE CLINICAL


SCIENCES STREM- MARCH 2008
OBSTETRICS AND GYNAECOLOGY
Q1
A 34 year primipara in labour at term has the baby in a
cephalic presentation. She has three uterine
contractions in ten minutes but has remained at 6cm
dilatation of the cervix for four hours. The membranes
ruptured ten hours ago. During the past one hour she
has a temperature of 40.00C.
1.1
Explain the clinical findings you would
obtain to determine the present condition of this labour
(50 marks)
1.2
1.2 Describe the complications you would
expect in the mother and the baby following delivery and
the pueperium. (50 marks)
Q2
A 28 year primigravida at a period of gestation of 32
weeks presented with vaginal bleeding
2.1 Name 04 possible causes for her bleeding per
vagina (10 marks)
2.2 Describe the diagnostic features of the above
conditions. (50 marks)
2.3 Describe the management of one of the above
conditions (40 marks)
Q3
A 35 yr old mother of one child was admitted today to
the gynaecology casuality ward with acute abdominal
pain of 04 hours duration. Her last regular menstrual
period was on 2nd February 2008. On examination she
has a lump arising from the pelvis compatible with a
fourteen week size pregnant uterus.
3.1 Name 04 possible causes for her presentation.
(10 marks)
3.2 Outline the immediate management of this
patient. (40 marks)
3.3 outline the clinical features you would elicit and
the investigations you would perform to arrive at a
diagnosis. (50 marks)
Q4
Outline the initial management of the following clinical
situations
4.1 A 26yr old mother of three children and her
husband requesting sterilization. (35 marks)
4.2 A primigravida has profuse bleeding fifteen
minutes after a normal delivery. Her uterus is well
contracted.
4.3 A primigravida who presents at twenty weeks
gestation with a haemoglobin level of 8.2 g/dl
(35marks)
END OF COURSE EXAMINATION OF THE CLINICAL
SCIENCES STREAM- DECEMBER 2010

FACULTY OF MEDICINE COLOMBO


OBSTETRICS AND GYNAECOLOGY-1
(1)A para 03 gravida 02 admitted at 34 weeks of
gestation as she is found to be having single fetus with
breech presentation. Her first and second pregnancy
ended up as normal vaginal deliveries.
1.1 Name 06 causes for her breech presentation with
explanation for each cause. (20 marks)
1.2 Mention how you would assess this pregnancy.
1.3 While inward she had pre-term, pre-labour rupture of
membrane at 35 of weeks of pregnancy. Outline the
steps in the immediate management and the rest of the
pregnancy up to delivery (50 marks)
(2) Outline briefly the pathophysiological basis of the
immediate clinical steps required by the attending
medical officer in the following two conditions at the
antenatal booking visits.
2.1 24 yr old primigravida with positive urinary bte hCG
test at 5 weeks POA taking thyroxin 100g/day
replacement for primary hypothyroidism and having a
serum TSH measurement of 5.0mU/l (normal 0.4-4)
(40 marks)
2.1 39 years old mother of three children taking
Metformin for type 2 DM since 2 years and using natural
method of contraception found to be pregnant at weeks
POA. (60 marks)
(3)A 48 year old mother of two children presents with an
abdominal mass that is of ovarian origin.
3.1 name 05 clinical features that would determine that
this mass is malignant (20 marks)
3.2 How would you prepare this patient for surgery if it is
likely to be malignant. (30 marks)
outline the immediate and long term management of this
patient (50 marks)
(4)Write notes on:
4.1 Non surgical treatment of endometriosis. (40
marks)
4.2 Importance of assessing fetal size in antenatal
care in the third trimester (30)
4.3 Counselling a 45 year old woman who is admitted
for a total abdominal hysterectomy
for
menorrhagia due to uterine fibroids. (40 marks)
Obstetrics & Gynaecology-1
1.1. List 05(five) indications for induction of labour
at term (15 marks)
1.2. Mention the obstetric assessment & give
expected finding that should be there for
induction of labour (15 marks)

1.3. Name two cervical ripening methods


mentioning advantages & disadvantages
Of each method (30 marks)
1.4. Describe the advantages & disadvantages
of& complications of induction of labour
(40
marks)
2) A 39 year old primigravida married 7 years with
anovular oligo-menorrhoea & hirsutism, has
conceived following ovulation induction with
clomiphene citrate. Her period of amenorrhoea is 8
weeks at the booking antenatal visit. She weight
85kg, has a sitting blood pressure of 50/100mmHg
& urine proteins are negative with reducing
substance being present.
2.1. Mention the most likely diagnosis for her
anovulation (10 marks)
2.2. Outline briefly the pathophysiological basis
for her current metabolic problem (30 marks)
2.3. List five fetal complications she is likely to
encounter. (15 marks)
2.4. Outline how would manage her as an
intern house officer in obstetrics & gynaecology
from antenatal booking up till delivery (45 marks)
3) A term baby born by a normal vaginal delivery
following an uneventful pregnancy is admitted on
Day 05 with poor sucking and fever,
3.1 list the important clinical features (history and
examination findings) that would help in the initial
evaluation of this neonates condition. (40 marks)
3.2 Mention the most likely diagnosis. (10 marks)
3.3 list the investigations you would perform on this
neonate, giving the expected findings. (30 marks)
3.4 list 05 features (clinical or laboratory) that would
indicate a poor prognosis (20 marks)
4) A term otherwise healthy, six weeks old infant is
brought to the Well baby clinic because his mother
thinks that she has inadequate breast milk
4.1 list the clinical features that would help you
decide whether there is lactation failure or not. (50
marks)
4.2 outline the feeding advice you would give a
mother of a healthy 06 months old infant who plans
to resume work as a school teacher in another
weeks time. Her total travel time is one hour per
day. The infant has been exclusively breast fed and
is thriving along the 50th centile. Include in your
answer, feeding advice until the age of one year.
(50 marks)

A PROJECT OF MSWS AND MFSU- COLOMBO (2010/2011)

SURGERY
FACULTY OF MEDICINE- COLOMBO
2008 DECEMBER
1) A 2 year old boy was admitted to the surgical casualty
ward with an incarcerated inguinal hernia.
1.1 Describe the clinical features of this condition.
(30 marks)
1.2 List the additional clinical features that will suggest
strangulation of the hernia contents (30 marks)
1.3 Outline the management of this child with an
incarcerated inguinal hernia. (40 marks)
2) A 38 year old woman complains of a breast lump of
two months duration.
2.1 List the clinical features that would indicate a breast
carcinoma. (30 marks)
2.2 Explain, giving reasons, the investigations that you
would request to arrive at a diagnosis. (30 marks)
2.3 It has been decided to perform a total mastectomy
and axillary clearance on this patient. (30 marks)
2.3 What information would you give this patient at the
time of obtaining consent for surgery? (40 marks)
3) A 65 year old man develops fever 5 days after right
hemi- colectomy.
3.1 List five possible causes for the fever. (10 marks)
3.2 Describe how you would clinically differentiate the
causes mentioned in 3.1 (30 marks)
3.3 List the investigations (giving reasons) you would
request. (30 marks)
3.4 Outline the management of two of the conditions
listed in 3.1. (30 marks)
4) Outline the advice you would give a patient going
home following under mentioned procedures.
4.1 Anterior resection for carcinoma rectum. (40 marks)
4.2 Colonoscopy and diathermy excision of a sigmoid
colon polyp under intra venous sedation. (30 marks)
4.3 Manipulation and application of a POP cast for a
Colles fracture. (30 marks)
5) Describe how you would clinically suspect or
diagnosed the following conditions.
5.1 Torsion of testis. (25 marks)
5.2 Benign enlargement of prostate. (25 marks)

14 | P a g e

5.3 Vesicle calculus. (25 marks)


5.4 Renal cell carcinoma. (25 marks)
6) A 24 year old man was admitted to the accident
service with a crush injury to the right leg. He has no
other injury.
6.1 On what clinical features and investigations would
you diagnosed a compound fracture of tibia. (40 marks)
6.2 Describe the management of this patient in the first
24 hours. (40 marks)
6.3 List the possible complications of this injury.
(20 marks)

FACULTY OF MEDICINE- COLOMBO


2009 DECEMBER
1) A 30 year old female presents with a solitary nodule
in the thyroid.
1.1 Describe the clinical features that suggest
malignancy. (30 marks)
1.2 What investigations would you carry out to assess
its spread and briefly describe the expected findings?
(30 marks)
1.3 Describe the long term management of a patient
who has undergone total thyroidectomy for malignancy
of the thyroid. (25 marks)
1.4 Describe the lymphatic drainage of the thyroid.
(15 marks)
2) A 20 year old male presents with a history of
dyspepsia of 2 years duration.
2.1 Describe the clinical features that would suggest
gastro oesophagial reflux disease (GORD) in this
patient. (30marks)
2.2 List, giving expected findings, the investigations that
you would carry to arrive at a diagnosis of GORD.
(20 marks)
2.3 Enumerate the complications of GORD in this
patient. (20 marks)
2.4 Outline the treatment of GORD. (30 marks)
3) A 25 year old man suffers multiple injuries from a
road traffic accident. Two hours later, his GCS is 10,
systolic BP 80mmHg. Pulse is 120/min and the
respiratory rate is 25/min.
3.1 List 3 possible mechanisms contributing to his GCS
of 10. (15 marks)

3.2 Outline the pathophysiology of his deranged


cardiovascular functions. (30 marks)
3.3 Explain how you would estimate the blood loss in
this man. (20 marks)
3.4 Outline how you would resuscitate this patient.
(35 marks)
4) A 60 year old female who underwent left
hemicoletomy for malignancy develops tachypnoea and
dyspnoea on the 5th post operative day. She gives a
past history of IHD. Her pulse oxymetry saturation is
90% on air.
4.1 List four possible causes for above clinical picture.
(10 marks)
4.2 Describe how you would differentiate the causes
listed in 4.1 (40 marks)
4.3 Explain the pathophysiology of her tachypnoea,
dyspnoea, and low pulse oxymetry saturation
(20marks)
4.4 Outline the treatment of one of the causes listed in
4.1 (30marks)
5) Write notes on diagnosis and treatment of;
5.1 Non retractable prepuce in 6 year old boy
(30 marks)
5.2 Urethral injuries (40 marks)
5.3 A 14 month old boy presented with an empty left
hemi- scrotum. (30 marks)
6) A 25 year old man has fallen from a height. He is
lying on ground complaining of backache.
6.1 Outline the first aid measures and how you would
transfer him to the nearest hospital. (20 marks)
The patient arrives at the accident service 2 hours later.
6.2 Describe the clinical features that would suggest a
spinal cord injury. (25marks)
6.3 Giving reasons, what investigations that you would
do to confirm your clinical diagnosis of spinal cord injury.
(25 marks)
6.4 Outline the principles of rehabilitator of a patient with
traumatic paraplegia (30marks)

FACULTY OF MEDICINE- COLOMBO


2009 DECEMBER
1. A 50 years old male is diagnosed as having a
malignant lesion in the lower oesophagus.

1.1List 4 malignant lesions that could be arise from the


oesophagus. (10 marks)
1.2Describe the pathological features in one lesion
listed in question 1.1
(20marks)
1.3List the investigations used staging oesophageal
malignancy and state how each investigation contribute
to staging.
(25 marks)
1.4Outline the protective preparation of this patient for
an oesophagectomy. (30 marks)
1.5Describe the lymphatic drainage of the oesophagus.
(15marks)
2. An 18 year old male with a history of appendicectomy
9 months back, presents with features of acute
abdomen obstruction.
2.1 Describe the clinical features that are likely to be
present in this patient.
(30 marks)
2.2 List 3 most likely causes for intestinal obstruction in
this patient. (10 marks)
Outline the pathophysiology and consequences of acute
intestinal obstruction. (30 marks)
2.3 outline the preoperative management of this patient.
(30 marks)
3) Describe the similarities and differences in the clinical
features of;
3.1 Anal fissure and a rectal polyp in a 3 year old child.
(25 marks)
3.2 Suppurative inguinal lymphadenitis and acute
incarcerated inguinal hernia in a 2 year old girl. (25
marks)
3.3 Testicular torsion and idiopathic scrotal oedema in a
6 year old boy. (25 marks)
3.4 Cellulitis acute osteomyelitis of the leg in a 3 year
old boy. (25 marks)
4) A 40 year old women with a family history of thyroid
cancer presents with a goiter.
4.1 Describe the clinical features that suggest
malignancy in this patients goiter. (40 marks)
After investigation, a decision has been taken to
carryout total thyroidectomy.
4.2 Outline the operative preparation of this patient. (30
marks)
4.3 Explain the pathological basis for total thyroidectomy
of this patient. (30 marks)
5) A 35 year old man presents with a large non healing
ulcer around the medial malleolar region of 6 months

A PROJECT OF MSWS AND MFSU- COLOMBO (2010/2011)

duration. He gives a history of femoral shaft fracture


following a road traffic accident and being bed ridden for
a long period. On examination, the leg is swollen and
has varicose veins. The peri- ulcer skin is dark.
5.1 What is the most likely mechanism for the leg ulcer
(25 marks)
5.2 List 3 reasons for non-healing of the ulcer.
(10 marks)
5.3 Outline how you would assess the reasons for non
healing of this ulcer. (20 marks)
5.4 Outline the treatment of this man. (30marks)
5.5 List the characteristics of an ideal wound dressing
(15 marks)
6) A 50 year old male labourer presents with law
backache of one week duration.
6.1 On what clinical features would you diagnosed
lumbar disk prolapse in this patient. (25 marks)
6.2 What is the pathophysiology of lumbar disc
prolapse? (15 marks)
6.3 List, giving reasons, the investigations you would
carryout on this patient to confirm the clinical diagnosis.
(20 marks).
6.4 Outline the treatment options (Also state indication/s
for each option) available for lumber dice prolapsed.
(40marks)

CLINICAL SCIENCES STREAM -2008


SURGERY
1)1.1. List four causes of post thyroidectomy stridor.
(20 marks)
1.2. Explain the causative mechanisms for each
cause stated in 1.1 (50 marks)
1.3 Outline the immediate management of a patient
with post thyroidectomy stridor (30 marks)
2) A 70 year old male was admitted to a surgical ward
with haematuria
2.1. Describe the clinical features & investigations
which will lead to diagnosis if vesical
calculus (40 marks)
2.2. List 4 other likely causes of haematuria in this
patient (10 marks)
2.3. Discuss the factors which are likely to contribute
to the formation of vesical calculus in this
patient (30 marks)
2.4. Outline the pre operative preparation of this
patient if he has to undergo vesicolithotomy
under general anaesthesia (20 marks)

15 | P a g e

3) Indicate the advice you would give discharged


following the under mentioned surgical procedures
3.1 Repair of inguinal hernia (20 marks)
3.2. Anterior resection for carcinoma of the rectum
(30 marks)
3.3 Permanent colostomy for benign disease (30
marks)
3.4 Colonoscopy under IV sedation (20 marks)

PAEDIATRICS
Faculty of Medicine -Colombo 2009
December
1) A ten year old girl complains of feeling faint 5
minutes after you have administrated rubella vaccine to
her, in a Child Welfare Clinic.
1.1 Mention two conditions that could give rise to her
symptoms. (10 marks)
1.2 Outline how you would differentiate between two
conditions you mentioned in 1.1 (30 marks)
Within the next ten minutes her general condition
deteriorated rapidly.
1.3 Describe in detail how you would manage this childs
clinical situation. (40 marks)
1.4 List the further actions you would take after dealing
with her medical management. (20 marks)
2) A seven year old boy is bitten on the foot by an
unidentified snake.
2.1 (a) Describe the first aid measures that are
indicated. (15 marks)
He is admitted to the hospital two hours after the
incident.
2.1 (b) Describe your management of this boy during
the initial 24 hours after admission. (40 marks)
A 3 year old boy was admitted to the casualty ward with
stridor of two hours duration. The grandmother who
admits the child is unaware of his immunization status.
2.2(a) List three possible causes for his symptoms.
(15 marks)
2.2(b) Outline the management of any one of the
conditions mentioned by you in 2.2(a) (30 marks)
3) A two year old girl is bought to the out patient
department with a history of fever of 12 hours duration.
She has a past history of one episode of simple febrile
convulsion at the age of 15 months.
3.1 Discuss your management of this child. (30 marks)
On the fourth day of illness she is brought back to you
with persisting fever. She has been vomiting and is
reported to be lethargic. On examination you notice that

she is ill looking and has an erythematous macular rash


on limbs and trunk.
3.2 (a) Mention two likely diagnosis (10 marks)
3.2(b) Outline your management of this child in this
present condition. (30 marks)
Six hours later, the vomitus contains coffee ground
material and the peripheral circulation is poor.
3.3 Describe your immediate management. (30 marks)

PAEDIATRICS

Faculty of Medicine Colombo- 2010


December
1) A nine year old boy is admitted with fever of three
weeks duration. He has a generalized erythematous
rash. That is fluctuating in severity and mild
hepatosplenomegaly. He looks ill; there is no
conjunctivitis, respiratory or urinary signs. He is the son
of a chena cultivator and lives in rural area. There has
been no response to oral amoxicillin given at the
nearest district hospital.
1.1 List three possible conditions in the differential
diagnosis of this childs illness. (15 marks)
1.2 Mention two clinical features associated with each of
the three conditions you have mentioned in 1.1.
(30 marks)
1.3 List the investigations that would help you to
establish the diagnosis giving the likely results in each
of the condition mentioned in 1.1 (30 marks)
1.4 Outline the treatment of any one condition you have
mentioned in 1.1 (25 marks)
2) 2.1 List the clinicpathological features helpful in
differentiating acute glomerulonephritis from minimal
change nephritic syndrome in childhood. (35 marks)
2.2 A ten year old is diagnosed to have acute
glomerulonephritis. Briefly discuss the involved
management of this child during the first 48 hours after
admission. (45 marks)
2.3 Enumerate 4 indications for a renal biopsy in an 11
year old with nephritic syndrome. (20marks)
3) (a) A 2.3Kg baby boy is born to a primigravide at
32weeks gestation by vaginal delivery.
3.1 List the problems you will encounter in managing
this baby in the first 72 hours. (25 marks)
3.2 The baby is noted to be grunting at 3 hours of age.
Enumerate the management in the next six hours.
(25 marks)
B. You are asked to come urgently to the labor room to
attend on a baby born at term to a mother with
gestational diabetes, and is weight 3.8Kg.
3.3 List the problems you would anticipate I this baby.
(25 marks)

3.4 How you would manage this baby in the first 48


hours. (25 marks)
4) (A) An 18 month old with a cyanotic heart disease
presents with fever of three days and a seizure on the
day of admission. On examination he looks drowsy and
ill.
4.1 List two causes that you would consider in the
differential diagnosis. (10 marks)
4.2 Explain the pathophysiology of each of the two
causes mentioned in 4.1 (30 marks)
B. A ten year old girl with beta thalassaemia major has
undergone spleenectomy six months ago. She presents
with fever, abdominal pain and distention, cold
peripheries and hypotension.
4.3 Give the most likely diagnosis with reasons.
(30 marks)
4.4 Describe the immediate management of this patient
(30 marks)
5) You are called to see an8 month old infant I the
emergency department who presents with vomiting and
loose stools of one days duration. The nurse reports that
the child appears Dehydrate
5.1 describe the clinical features that would help you
decide o the degree of dehydration. (30marks)
The child is admitted and treated with oral rehydration
solution (Jeewanee)
5.2Write the mechanism of action of oral rehydration
solution stating the composition of the product. ( acw
osmolality formula) That is available in the state
hospitals in Sri Lanka. (30 marks)
5.3 There is blood in the stools noticed the next morning
.Outline your management. (Mentioning the differential
diagnosis, clinical features, investigations and treatment
options.) (40 marks)

sensation in a glove and stocking


distribution in his right arm and leg.
1.1 What are the most likely diagnoses?
(10)
1.2 Explains the terms of primary gain
and secondary gain (30)
1.3 Describe the management of this
patient.(60)
2.

Describe briefly how you would manage the


following patients.
2.1 A 28 year old woman who has
delivered a healthy baby one week ago
refuses to feed or care for him
because she hears voices telling that
the baby is abnormal. (40)
2.2 A 48 year old man who keeps abusing
his wife about assumed acts of sexual
infidelity. (30)
2.3 A 39 year old man attends a medical
clinic, you note also to be a heavy
smoker.(30)

3.

Discuss the pharmacological management of


the following.
3.1 A 35 year old house wife has panic
disorder. She gets several panic
attacks per day and is very distressed
by it. There is no associated
agoraphobia or depressive disorder.
(30)
3.2 An 18 year old boy is admitted with a
first episode of mania. He has poor
sleep, is disinhibited and disturbs other
patients in the ward. (40)
3.3 A 70 year old retired accountant is
diagnosed with dementia. He has poor
memory and scores 18/30 in the
MMSE. There is no disturbance of
behavior. He does not have secondary
medical cause for Dementia. (30)

4.

4.) A 45 year old man is admitted to a


general medical ward with a history of
disturbed behavior for a few days duration.
Subsequent inquiries reveal that he was
suffering from a long term mental illness with
poor drug compliance. The aging parents,
both pensioners are very concerned about

PSYCHIATRY
FACULTY OF MEDICINE -COLOMBO
END OF COURSE EXAMINATION OF THE CLINICAL
SCIENCES STREAM
(MAIN)DECEMBER2009
PSYCHOLOGICAL MEDICINE 1 (ESSAY)
Answer all questions (3hrs)
1.
A 20 year old sailor is referred for treatment.
One week back after an argument with his
superior officer his right arm and left leg
became suddenly paralysed. On examination
his reflexes bilaterally are normal but there is
partial loss of power and complete loss of

A PROJECT OF MSWS AND MFSU- COLOMBO (2010/2011)

16 | P a g e

the patients inability to engage in work,


social isolation and lack of regular income.
4.1 Give four reasons for the lack of drug compliance in this
patient. (20)
4.2 What options available to improve his drug compliance.
(30)
4.3 Describe briefly the services available to meet his
psychosocial needs mentioned above. (50)
5.) An 8 year old boy is referred to you by an MOH in a rural
area for difficulties with school work. He was born
premature with a birth weight of 1.2 kg. His early motor and
language milestones were delayed. Outline the following.
5.1 Assessment to identify health (physical, mental and
social) needs.( 30)
5.2 Assessment to identify educational needs. (30)
4.4 Preventive measures that could have been taken in
early development in this child.(40)

1.2
1.3

2.1
2.2
2.3

2.
List five features of depressive disorder. (15)
Describe briefly the management of a 45 year
old man diagnosed for the first time with a
depressive disorder. (35)
Describe in detail the risk assessment of a 35
year old woman brought to the OPD having been
discovered just prior to swallowing a large
number of paracetamol tablets.(50)
3.

FACULTY OF MEDICINE-COLOMBO
END OF COURSE EXAMINATION OF THE CLINICAL
SCIENCES STREAM
(MAIN)DECEMBER2010
PSYCHOLOGICAL MEDICINE 1 (ESSAY)
Answer all questions (3hrs)
1.
A 26 year old woman is diagnosed with
schizophrenia. She is currently on
Trifluperazine 10mg daily. She has had
three relapses of her illness in the last four
years.
1.1 Write five symptoms known to occur as side
effects of her medication. (15)

Describe the advantages and disadvantages of


treating her with Olanzapine instead of
Trifluperazine. (35)
Describe in detail the advice you will give if she
says she intends to stop her medication as she
is due to get married next month and does not
want her husband to know of her illness. (50)

3.1
3.2
3.3
3.4

A 60 year old retired is referred to you for


memory loss. On cognitive testing you find
that Mini Mental State Examination Score
(MMSE) is 23 out of 30 with significant
impairment in short term memory. An MRI
scan reports there is cerebral atrophy but
no small vessel disease or evidence of a
space occupying lesion.
List the other investigations you would do in this
patient.(20)
If the tests in 3.1 are negative what is the most
likely diagnosis?(10)
Write two drugs effective for this condition;
describe their dose regimes and common side
effects.(30)
What advice would you give the immediate
relatives/carers of this patient(40)

4.1
4.2

4.3

4.
Outline the message you will give a group of
Municipal Council labourers attending a tobacco
smoking prevention programme. (30)
What steps will you take to help a 45 year old
house wife with two children cope with a
diagnosis of breast cancer made one week ago?
(30)
Describe briefly the psychological management
in a 30 year old man diagnosed with recurrent
panic attacks.(40)
5.

A 9 year old boy with poor school


performance is diagnosed to have ADHD
following
clinical
psychometric
assessments.
5.1 Describe prominent behaviors likely to be given
in the history by the mother and class teacher
that would support the diagnosis. (40)
5.2 List three co-morbid disorders that you would
look for. (15)
5.3 The boy is treated with methylphenidate.
Describe how you would monitor treatment
during follow up. (15)
5.4 Describe
the
behavioral
interventions you could do with
this boy. (30)

A PROJECT OF MSWS AND MFSU- COLOMBO (2010/2011)

17 | P a g e

Você também pode gostar